Download as pdf or txt
Download as pdf or txt
You are on page 1of 64

Total Marks : 200

Online Prelims TEST - 10 (SUBJECT WISE)


( InsightsIAS Mock Test Series for UPSC Preliminary Exam 2020 ) Mark Scored : 98.67

1 With reference to the Gross Domestic Product (GDP) deflator, consider the following statements:
1. It is used to measure the inflation in the country.
2. It does not include the price of imports.
3. The base year for GDP deflator is 2011-12.

Which of the following statements given above is/are correct?


A. 3 only
B. 1 and 2 only
C. 1 and 3 only
D. 1, 2 and 3

Your Answer :
Correct Answer : B

Answer Justification :
4
03inflation. Hence,
The Gross Domestic Product (GDP) deflator is a measure of general price
1
statement 1 is correct. 64
7 54
-8
It is calculated by dividing nominal GDP by real GDP and then multiplying by 100. Nominal GDP is
the market value of goods and services produced in an economy,
. c om unadjusted for inflation (It is the
in real output (It is the GDP measured at constant m
a l adjusted for inflation to reflect changes
GDP measured at current prices). Real GDP is nominal iGDP,
prices).
2 @g
m2 Consumer Price Index (CPI) and Wholesale Price
There are other measures of inflation too like
a
Index (or WPI); however GDP deflator p is a much broader and comprehensive measure. Since Gross
umeasure
Domestic Product is an aggregate a n of production, being the sum of all final uses of goods
a s
and services (less imports),
- d GDP deflator reflects the prices of all domestically produced goods and
services in the economy s whereas, other measures like CPI and WPI are based on a limited basket of
goods and services,Dathereby not representing the entire economy (the basket of goods is changed to
accommodate p amchanges in consumption patterns, but after a considerable period of time). Another
u
An distinction is that the basket of WPI (at present) has no representation of services sector.
important

The GDP deflator also includes the prices of investment goods, government services and
exports, and excludes the price of imports. Hence, statement 2 is correct.

Changes in consumption patterns or the introduction of new goods and services or structural
transformation are automatically reflected in the deflator which is not the case with other inflation
measures.
However WPI and CPI are available on monthly basis whereas deflator comes with a lag (yearly or
quarterly, after quarterly GDP data is released). Hence, monthly change in inflation cannot be
tracked using GDP deflator, limiting its usefulness.

Ministry of Statistics and Programme Implementation (MOSPI) comes out with GDP deflator in
National Accounts Statistics as price indices. The base of the GDP deflator is revised when base of
GDP series is changed. The latest available GDP deflator series with 2004-05. Hence,
statement 3 is incorrect.

prelims.insightsonindia.com 1
© Insights Active Learning | All rights reserved - 131815. You may not reproduce, distribute or exploit the contents in any form without
written permission by copyright owner. Copyright infringers may face civil and criminal liability
Total Marks : 200
Online Prelims TEST - 10 (SUBJECT WISE)
( InsightsIAS Mock Test Series for UPSC Preliminary Exam 2020 ) Mark Scored : 98.67

2 Which of the following curve depicts the relationship between distribution of income and inequality in
an economy?

A. Philips Curve
B. Laffer Curve
C. Lorenz Curve
D. Kuznet Curve

Your Answer : C
Correct Answer : C

Answer Justification :

The Lorenz curve is a graphical representation of income inequality or wealth inequality developed
by American economist Max Lorenz in 1905. The graph plots percentiles of the population on the
horizontal axis according to income or wealth. Hence, option (c) is correct.
4
1 03
64
54 economy?
3 Which of the following items does/do form the factors of production in a capitalist
7
1. Land
-8
2. Labour
. c om
3. Capital
a il
4. The enterprise m
2 @g
2 below:
Select the correct answer using the code given
m
a
A. 1 and 2 only
n up
B. 3 and 4 only
a sa
-d
C. 1, 2 and 3 only
D. 1, 2, 3 and 4 s
Da
p
Your Answer a:mD
u
AnAnswer : D
Correct

Answer Justification :

Factors of production is an economic term that describes the inputs used in the production of goods
or services in order to make an economic profit. These include any resource needed for the creation
of a good or service. The factors of production include land, labor, capital, and
entrepreneurship. Hence, option (d) is correct.

prelims.insightsonindia.com 2
© Insights Active Learning | All rights reserved - 131815. You may not reproduce, distribute or exploit the contents in any form without
written permission by copyright owner. Copyright infringers may face civil and criminal liability
Total Marks : 200
Online Prelims TEST - 10 (SUBJECT WISE)
( InsightsIAS Mock Test Series for UPSC Preliminary Exam 2020 ) Mark Scored : 98.67

4 Which of the following is/are form a part of revenue expenditure?


4
1. Salaries
1 03
2. Defence expenditure
4 64
5
3. Major Subsidies
4. Expense towards new infrastructure projects - 87
.c om
Select the correct answer using the code given below.ail
A. 1, 3 and 4 only
@ gm
B. 1, 2 and 3 only 2 2
C. 1, and 3 only
p am
D. 1, 2, 3 and 4 n u
s a
Your Answer : C - da
Ba
Correct Answer : D
s
p am
Answer u
Justification :
An
Revenue Expenditure is expenditure incurred for purposes other than the creation of physical or
financial assets of the central government. It relates to those expenses incurred for the normal
functioning of the government departments and various services, interest payments on
debt incurred by the government, and grants given to state governments and other parties
(even though some of the grants may be meant for creation of assets).

While capital expenditure or capex is a broad economic term used by several companies, it is a very
important term in a budgetary context.

It usually denotes funds used by a company (government) to acquire, maintain or upgrade


physical assets such as property, new infrastructural projects or buying new equipment.

Capital expenditure is classified as a long-term expenditure and usually includes expenses incurred
by the government towards asset building, including developmental and infrastructural projects.

When a government spends money on big-ticket projects, the expenses incurred are usually

prelims.insightsonindia.com 3
© Insights Active Learning | All rights reserved - 131815. You may not reproduce, distribute or exploit the contents in any form without
written permission by copyright owner. Copyright infringers may face civil and criminal liability
Total Marks : 200
Online Prelims TEST - 10 (SUBJECT WISE)
( InsightsIAS Mock Test Series for UPSC Preliminary Exam 2020 ) Mark Scored : 98.67

categorised as capital expenditure. Such expenses are not recurring in nature.

4
1 03
4 64
5
Hence, option (b) is correct. - 87
.c om
i l
https://www.indiatoday.in/budget-2019/budget-faqs/story/union-budget-2019-guide-to-key-terms-fre
ma
quently-asked-questions-glossary-1551985-2019-06-19
g
@
m 22
5 Which of the following constitute ForeignaExchange
u p Reserves of India?
n
sa
1. Foreign currency assets
a
-d
2. Diamond
3. Gold s
Da
4. Special Drawing Rights
p am
Select nu correct answer using the code given below.
the
A
A. 1, 2 and 3 only
B. 2 and 3 only
C. 1 and 4 only
D. 1, 3 and 4 only

Your Answer : D
Correct Answer : D

Answer Justification :

India's forex reserves comprise Foreign Currency Assets (FCAs), gold reserves, Special
Drawing Rights (SDRs) and India's reserve position with the International Monetary Fund (IMF).

FCAs are the largest component of the forex reserves.

Hence, option (d) is correct.

prelims.insightsonindia.com 4
© Insights Active Learning | All rights reserved - 131815. You may not reproduce, distribute or exploit the contents in any form without
written permission by copyright owner. Copyright infringers may face civil and criminal liability
Total Marks : 200
Online Prelims TEST - 10 (SUBJECT WISE)
( InsightsIAS Mock Test Series for UPSC Preliminary Exam 2020 ) Mark Scored : 98.67

https://www.livemint.com/market/stock-market-news/india-s-forex-reserves-up-by-1-3-billion-to-over-
420-billion-1558155104850.html

6 With reference to Union Budget, which of the following is/are covered under Capital receipts?
1. Disinvestment
2. Interest and dividend on government investment
3. Funds from Public Provident Fund
4. Revenue from Income tax

Select the correct answer using the code given below.


A. 1 and 3 only
B. 2, 3 and 4 only
C. 1 and 4 only
D. 1, 2, 3 and 4

4
Your Answer : A
1 03
64
Correct Answer : A
5 4
Answer Justification :
- 87
om
.cborrowing) or (ii) reduce assets (e.g.
Government receipts which either (i) create liabilities (e.g.
i l
disinvestment) are called capital receipts. Thus whenagovt. raises funds either by incurring a
m
@agcapital receipt.
liability or by disposing off its assets, it is called
2
m 2
a create liability are Borrowing and raising of funds from
Two examples of Capital Receipts which
p
Public Provident Fund and Small u
n savings deposits.
s a
da
Borrowings are treated -capital receipts because they create liability of returning loans. Similarly,
funds raised fromD as small saving deposits in post offices and banks are treated capital receipts
PPF,
because they mIncrease liability of the government to repay these amounts to PPF (Public Provident
p a
u and small savings depositors.
Fund) holders
An
Two examples of Capital Receipts which reduce assets are Disinvestment and Recovery of
Loans. Disinvestment by government means selling a part or whole of its shares of public sector
undertakings. Funds raised from disinvestment reduce government assets

Recovery of loan is also capital receipt as it reduces government assets.

Government receipts which neither (i) create liabilities nor (ii) reduce assets are called revenue
receipts. These are proceeds of taxes, interest and dividend on government investment,
cess and other receipts for services rendered by the government.

Hence, option (a) is correct.

7 Stagflation can lead to which of the following in the economy?


1. Inflation
2. Low level of unemployment

prelims.insightsonindia.com 5
© Insights Active Learning | All rights reserved - 131815. You may not reproduce, distribute or exploit the contents in any form without
written permission by copyright owner. Copyright infringers may face civil and criminal liability
Total Marks : 200
Online Prelims TEST - 10 (SUBJECT WISE)
( InsightsIAS Mock Test Series for UPSC Preliminary Exam 2020 ) Mark Scored : 98.67

3. High economic growth

Select the correct answer using the code given below.


A. 1 only
B. 1 and 3 only
C. 1 and 2 only
D. 1, 2 and 3

Your Answer : C
Correct Answer : A

Answer Justification :

Stagflation is a condition of slow economic growth and relatively high unemployment, or


economic stagnation, accompanied by rising prices, or inflation. It can also be defined as
inflation and a decline in gross domestic product (GDP).
4
1 03
Hence, option (a) is correct.
4 64
5
https://www.investopedia.com/terms/s/stagflation.asp
- 87
.c om
il
8 With reference to Gross Value Added (GVA), consider theafollowing statements:
m
@g
1. GVA at basic prices will include production subsidies.
2
2. GVA at factor cost includes no taxes. 2
p am
u
an
Which of the statement given above is/are correct?
s
da
A. 1 only
B. 2 only
s -
C. Both 1 and D2 a
D. Neither
p am1 nor 2
nu
YourAAnswer :
Correct Answer : B

Answer Justification :

Gross value added (GVA) is defined as the value of output less the value of intermediate
consumption. Value added represents the contribution of labour and capital to the production
process. When the value of taxes on products (less subsidies on products) is added, the sum of value
added for all resident units gives the value of gross domestic product (GDP).

GVA at basic prices will include production taxes and exclude production subsidies
available on the commodity. Hence, statement 1 is incorrect.

On the other hand, GVA at factor cost includes no taxes and excludes no subsidies and GDP
at market prices include both production and product taxes and excludes both production and
product subsidies. Hence, statement 2 is correct.

prelims.insightsonindia.com 6
© Insights Active Learning | All rights reserved - 131815. You may not reproduce, distribute or exploit the contents in any form without
written permission by copyright owner. Copyright infringers may face civil and criminal liability
Total Marks : 200
Online Prelims TEST - 10 (SUBJECT WISE)
( InsightsIAS Mock Test Series for UPSC Preliminary Exam 2020 ) Mark Scored : 98.67

By formula: GVA at factor cost + (Production taxes less Production subsidies) = GVA at basic prices.

GDP at market prices = GVA at basic prices + Product taxes- Product subsidies

In the revision of National Accounts statistics done by Central Statistical Organization (CSO)
in January 2015, it was decided that sector-wise wise estimates of Gross Value Added (GVA) will
now be given at basic prices instead of factor cost.

http://www.arthapedia.in/index.php%3Ftitle%3DGross_Value_Added_(GVA)_at_basic_prices_and_GV
A_at_Factor_Costs

9 An increase in tax to GDP ratio of a country may indicate which of the following?
1. Growing economy.
2. Efficient tax administration

Select the correct answer using the codes given below: 4


A. 1 only
1 03
B. 2 only
4 64
5
C. Both 1 and 2
D. Neither 1 nor 2 - 87
.com
Your Answer : B a i l
Correct Answer : C g m
2 2@
Answer Justification : am
n up
a tax collection should also increase. If the economy is growing and
Ideally with increase in GDP, sthe
a
s -d
business is doing well, naturally, profits will be better and therefore taxes should also be higher.
Hence, statement a 1 is correct.
D
am
p administration helps in higher tax collection and reduces the tax evasion. Hence,
Efficient tax
n u
A
statement 2 is correct.

10 Which of the following is/are part of the Personal Disposable Income?


1. Non-tax Payments such as fines
2. Corporate Tax.
3. Personal Tax Payments.
4. Net Interest payments made by households

Select the correct answer using the code given below.


A. 1, 2 and 4 only
B. 3 only
C. 1, 2, 3 and 4
D. None

Your Answer : D
Correct Answer : D

prelims.insightsonindia.com 7
© Insights Active Learning | All rights reserved - 131815. You may not reproduce, distribute or exploit the contents in any form without
written permission by copyright owner. Copyright infringers may face civil and criminal liability
Total Marks : 200
Online Prelims TEST - 10 (SUBJECT WISE)
( InsightsIAS Mock Test Series for UPSC Preliminary Exam 2020 ) Mark Scored : 98.67

Answer Justification :

None of them are part of Personal Disposable Income.

In economics, personal income refers to an individual's total earnings from wages, investment
enterprises, and other ventures. It is the sum of all the incomes received by all the individuals or
household during a given period.

By formula it can be given as:

Personal Income (PI) ≡ NI – Undistributed profits – Net interest payments made by households
– Corporate tax + Transfer payments to the households from the government and firms.

If we deduct the Personal Tax Payments (income tax, for example) and Non-tax Payments (such as
fines) from PI, we obtain what is known as the Personal Disposable Income.

Personal Disposable Income (PDI ) ≡ PI – Personal tax payments – Non-tax payments.


4
1 03
Hence, option (d) is correct.
4 64
5
87
- Institutions (SIFIs)
11 Consider the statements regarding Systemically Important Financial
m
1. The idea of SIFI has emerged from the Pittsburgh summit
i l .cofoWorld Bank.
ma perceived as institutions that are Too Big
2. Systemically Important Financial Institutions (SIFIs) are
g
to Fail (TBTF). @ 2
m 2
a correct?
Which of the statements given abovepis/are
u
A. 1 only
s an
B. 2 only d a
C. Both 1 and 2 s -
Da2
D. Neither 1 nor
p am
u :
An
Your Answer
Correct Answer : B

Answer Justification :

Systemically Important Financial Institutions (SIFIs) as institutions “whose distress or disorderly


failure, because of their size, complexity and systemic interconnectedness, would cause significant
disruption to the wider financial system and economic activity”.

At global level, based on the suggestion of G-20 Leaders in Pittsburgh summit in 2009, FSB
spearheads the efforts of formulating a framework for assessing and regulating SIFIs. Hence
Statement 1 is incorrect.

Systemically Important Financial Institutions (SIFIs) are perceived as institutions that are Too Big
to Fail (TBTF). Hence Statement 2 is correct.

12 When the Reserve Bank of India increases the repo rate by 50 basis points, which of the following likely to happen?

prelims.insightsonindia.com 8
© Insights Active Learning | All rights reserved - 131815. You may not reproduce, distribute or exploit the contents in any form without
written permission by copyright owner. Copyright infringers may face civil and criminal liability
Total Marks : 200
Online Prelims TEST - 10 (SUBJECT WISE)
( InsightsIAS Mock Test Series for UPSC Preliminary Exam 2020 ) Mark Scored : 98.67

A. India's GDP growth rate increases drastically


B. Foreign Institutional Investors may bring more capital into our country
C. It may drastically reduce the liquidity to the banking system.
D. Scheduled Commercial Banks may increase their lending rates

Your Answer : D
Correct Answer : D

Answer Justification :

Increase in repo rate by RBI would largely lead to increase in lending rates of banks.

13 Which of the following measures can be adopted to promote the Fiscal


Consolidation?
1. Increasing subsidies
2. Increasing the tax base 4
3. Supporting export oriented industries 1 03
4 64
5
Select the correct answer using the codes given below.
- 87
om
A. 2 and 3 only
B. 1 and 3 only
il . c
C. 2 only a
D. 1, 2 and 3 gm @
2 2
m
Your Answer : A
u pa
Correct Answer : A
a n
s
Answer Justifications :-
da
Da
am refers to the policies undertaken by Governments (national and sub-national
Fiscal Consolidation
p
levels) toureduce their deficits and accumulation of debt stock.
An
Increasing the subsidies increases the expenditure of government. Thus, it will go against the
principle of fiscal consolidation. Hence, statement 1 is incorrect.

Increasing the tax base increases the tax collection. Thus, it increases the revenue for the
government and promotes the fiscal consolidation. Hence, statement 2 is correct.

Supporting the export oriented industries help in increasing the forex reserves of the country. Thus,
help in promoting the fiscal consolidation. Hence, statement 3 is correct.

http://www.arthapedia.in/index.php?title=Fiscal_Consolidation

14 Which of the following measures is/are examples of expansionary fiscal policy?


1. Increasing the subsidies
2. Tax rebate to companies
3. Providing loans at higher interest rates

prelims.insightsonindia.com 9
© Insights Active Learning | All rights reserved - 131815. You may not reproduce, distribute or exploit the contents in any form without
written permission by copyright owner. Copyright infringers may face civil and criminal liability
Total Marks : 200
Online Prelims TEST - 10 (SUBJECT WISE)
( InsightsIAS Mock Test Series for UPSC Preliminary Exam 2020 ) Mark Scored : 98.67

Select the correct answer using the codes given below


A. 2 only
B. 1 and 2 only
C. 1 and 3 only
D. 1, 2 and 3

Your Answer : B
Correct Answer : B

Answer Justification :

Expansionary fiscal policy is a form of fiscal policy that involves decreasing taxes, increasing
government expenditures or both, in order to fight recessionary pressures.

Increasing the subsidies decreases the out of pocket expenditure. Thus, increases the spending
capacity. Hence, statement 1 is correct.

0 34
Tax rebate to companies increase the profit to companies. This further helps in
6 41 reducing the prices
of goods and services. Thus, increase the demand in the economy. Hence,
7 54 statement 2 is correct.
8
Providing loans at higher interest rate decreases the demand for -the loans. Thus, it decreases the
m
ocapacity.
liquidity in the market and leads to decreases the spending
i l . c Hence, statement 3 is

ma
incorrect.
g
2 2@
https://www.businessnewsdaily.com/3484-fiscal-policy.html
m
u pa
n
a sa
15 With reference to the Transfer payments, consider the following statements:

- d which the residents of a country receive ‘for free’.


1. Transfer payments are receipts
s
D a
2. They consist of remittances, gifts and grants.
3. They could bem given by the government or by private citizens living abroad
p a
n uthe
WhichA of statements given above is/are correct?
A. 1 and 2 only
B. 3 only
C. 1 and 3 only
D. 1, 2 and 3

Your Answer : D
Correct Answer : D

Answer Justification :

Transfer payments are the receipts which the residents of a country get for ‘free’, without
having to provide any goods or services in return. Hence, statement 1 is correct.

They consist of gifts, remittances and grants. Hence, statement 2 is correct.

They could be given by the government or by private citizens living abroad. Hence, statement

prelims.insightsonindia.com 10
© Insights Active Learning | All rights reserved - 131815. You may not reproduce, distribute or exploit the contents in any form without
written permission by copyright owner. Copyright infringers may face civil and criminal liability
Total Marks : 200
Online Prelims TEST - 10 (SUBJECT WISE)
( InsightsIAS Mock Test Series for UPSC Preliminary Exam 2020 ) Mark Scored : 98.67

3 is correct.

16 Consider the following statements:


1. The new series of national income was started in 2015 with the new base year 2013-14.
2. In its revision in January 2015 the CSO replaced GDP at factor cost with the GVA at basic prices.

Which of the statements given above is/are correct?


A. 1 only
B. 2 only
C. Both 1 and 2
D. Neither 1 nor 2

Your Answer :
Correct Answer : B

4
Answer Justification :
1 03
64
4 for GDP calculation to
From January 2015, the Central Statistics Office (CSO) updated base year
7 5
2011-12, replacing the old series base year of 2004-05, as per8the recommendations of
the National Statistical Commission. Hence, statement 1 is m
-
incorrect.

i l .co
ma
In India, the most highlighted measure of national income has been the GDP at
factor cost. The Central Statistics Office (CSO) ofgthe Government of India has
been reporting the GDP at factor cost and 2
@
at2market prices. In its revision in
m
January 2015 the CSO replaced GDP
u pa at factor cost with the GVA at basic prices, and the
GDP at market prices, which is now
a n called only GDP, is now the most highlighted measure. Hence,
statement 2 is correct. a s
s -d
Dastatements is/are correct regarding Net National Product (NNP)?
m
17 Which of the following
a
p
1. The newuseries of national income was started in 2015 with the new base year 2013-14.
n
2. In itsArevision in January 2015 the CSO replaced GDP at factor cost with the GVA at basic prices.

Which of the statements given above is/are correct?


A. 1 only
B. 2 only
C. Both 1 and 2
D. Neither 1 nor 2

Your Answer :
Correct Answer : A

Answer Justification :

If we deduct depreciation from GNP the measure of aggregate income that we obtain is called Net
National Product (NNP).

Thus NNP ≡ GNP – Depreciation.

prelims.insightsonindia.com 11
© Insights Active Learning | All rights reserved - 131815. You may not reproduce, distribute or exploit the contents in any form without
written permission by copyright owner. Copyright infringers may face civil and criminal liability
Total Marks : 200
Online Prelims TEST - 10 (SUBJECT WISE)
( InsightsIAS Mock Test Series for UPSC Preliminary Exam 2020 ) Mark Scored : 98.67

Hence, statement 1 is correct.

It is to be noted that all these variables are evaluated at market prices.

NNP at factor cost = National Income (NI ) = NNP at market prices – (Indirect taxes –
Subsidies) =NNP at market prices – Net indirect taxes (Net indirect taxes = Indirect taxes –
Subsidies). Hence, statement 2 is incorrect.

18 Reserve Bank of India (RBI) is banker to government. Other than lending to the government, which of
the following are the functions of the RBI?
1. It introduces and upgrades safe and efficient modes of payment systems in the country to meet the
requirements of the public.
2. It manages foreign exchange reserve
3. It performs a wide range of promotional functions to support national objectives.

Select the correct answer using the code given below 4


A. 2 only 1 03
B. 2 and 3 only
4 64
5
C. 1 and 2 only
D. 1, 2 and 3 - 87
.com
Your Answer : D a i l
Correct Answer : D g m
2 2@
Answer Justification : am
up
Main Functions of RBI s an
- da
as
Monetary Authority:
D
p am
A nu
Formulates, implements and monitors the monetary policy.

Objective: maintaining price stability while keeping in mind the objective of growth.

Regulator and supervisor of the financial system:

Prescribes broad parameters of banking operations within which the country's banking and
financial system functions.

Objective: maintain public confidence in the system, protect depositors' interest and provide
cost-effective banking services to the public.

Manager of Foreign Exchange

prelims.insightsonindia.com 12
© Insights Active Learning | All rights reserved - 131815. You may not reproduce, distribute or exploit the contents in any form without
written permission by copyright owner. Copyright infringers may face civil and criminal liability
Total Marks : 200
Online Prelims TEST - 10 (SUBJECT WISE)
( InsightsIAS Mock Test Series for UPSC Preliminary Exam 2020 ) Mark Scored : 98.67

Manages the Foreign Exchange Management Act, 1999.

Objective: to facilitate external trade and payment and promote orderly development and
maintenance of foreign exchange market in India.

Issuer of currency:

Issues and exchanges or destroys currency and coins not fit for circulation.

Objective: to give the public adequate quantity of supplies of currency notes and coins and in
good quality.

Developmental role 4
1 03
4 64
75 objectives.
Performs a wide range of promotional functions to support national
-8
https://www.rbi.org.in/Scripts/AboutusDisplay.aspx#MFl.c
om
a i
g m
2 @
2 following measures can be used to reduce inflation?
19 In context of the Indian economy, which of the
m
1. Increasing Cash Reserve Ratio (CRR). a
n up(SLR).
2. Increasing Statutory Liquidity Ratio
3. Reducing Reverse Repo Rate. a sa
s -d
Select the correctD a
answer using the code below
A. 1 and a m
2 only
u p
n 2 and 3
B. 1,
A
C. 2 and 3 only
D. 1 only

Your Answer : A
Correct Answer : B

Answer Justification :

All the above measures helps to reduce money supply in the market. Thus reducing inflation.

20 Qualified Foreign Investors (QFIs) are allowed to invest in which of the following segments of the
capital market?
1. Corporate bonds
2. Commercial Papers
3. Mutual funds

prelims.insightsonindia.com 13
© Insights Active Learning | All rights reserved - 131815. You may not reproduce, distribute or exploit the contents in any form without
written permission by copyright owner. Copyright infringers may face civil and criminal liability
Total Marks : 200
Online Prelims TEST - 10 (SUBJECT WISE)
( InsightsIAS Mock Test Series for UPSC Preliminary Exam 2020 ) Mark Scored : 98.67

Select the correct answer using the code given below


A. 1 and 2 only
B. 3 only
C. 2 and 3 only
D. 1, 2 and 3

Your Answer : D
Correct Answer : D

Answer Justification :

The Qualified Foreign Investor (QFI) is sub-category of Foreign Portfolio Investor and refers to any
foreign individuals, groups or associations, or resident, however, restricted to those from a country
that is a member of Financial Action Task Force (FATF) or a country that is a member of a group
which is a member of FATF and a country that is a signatory to International Organization of
Securities Commission’s (IOSCO) Multilateral Memorandum of Understanding (MMOU).
0 34
41a demat account in
QFIs are allowed to make investments in the following instruments by opening
6
any of the SEBI approved Qualified Depository Participant (QDP):
7 54
-8
Equity and Debt schemes of Indian mutual funds, . c om
a il
@ gm
2 2
Equity shares listed on recognized stock exchanges,
m
u pa
n
sa public offers
Equity shares offered through
a
s -d
Corporate D a listed/to be listed on recognized stock exchanges
bonds

p am
nu
AG-Securities, T-Bills and Commercial Papers

QFIs do not include FIIs/Sub-accounts/ Foreign Venture Capital Investor (FVCI).

Hence, option (d) is correct.

http://www.arthapedia.in/index.php?title=Qualified_Foreign_Investors_(QFIs)

21 With reference to the ‘Cash Management Bills’, consider the following statements:
1. They are the long term bills
2. The bills are issued by the RBI on behalf of the government
3. They are eligible as SLR securities for Banks.

Which of the following statements given above is/are correct?


A. 1 only
B. 2 and 3 only
prelims.insightsonindia.com 14
© Insights Active Learning | All rights reserved - 131815. You may not reproduce, distribute or exploit the contents in any form without
written permission by copyright owner. Copyright infringers may face civil and criminal liability
Total Marks : 200
Online Prelims TEST - 10 (SUBJECT WISE)
( InsightsIAS Mock Test Series for UPSC Preliminary Exam 2020 ) Mark Scored : 98.67

C. 1 and 3 only
D. 1, 2 and 3

Your Answer : B
Correct Answer : B

Answer Justification :

Cash Management Bills (CMBs) are short term bills issued by central government to meet its
immediate cash needs. Hence, statement 1 is incorrect.

The bills are issued by the RBI on behalf of the government. Hence the CMBs are short-term
money market instruments that help the government to meet its temporary cash flow
mismatches. Hence, statement 2 is correct.

CMBs are eligible as SLR securities. Investment in CMBs is also recognized as an eligible
of4the Banking
investment in Government securities by banks for SLR purpose under Section 24 3
Regulation Act, 1949. Hence, statement 3 is correct. 1 0
6 4
7
https://www.indianeconomy.net/splclassroom/what-are-cash-management-bills-cmbs/ 54
-8
. c om
il are promoted by Securities and
22 With reference to the Capital Market, which of the following
a
Exchange Board of India (SEBI)?
@ gm
1. Currency Futures
2 2
2. Insider trading m
3. Venture Capital Market u pa
4. Initial Public Offering (IPO) san
- da
as using the code given below.
Select the correct answer
D
A. 1, 2 andm 3 only
B. 1, 2 p a
nu and 4 only
C.A1, 3 and 4 only
D. 2, 3 and 4 only

Your Answer : C
Correct Answer : C

Answer Justification :

Currency futures are a transferable futures contract that specifies the price at which a currency can
be bought or sold at a future date. Currency futures contracts are legally binding and
counterparties that are still holding the contracts on the expiration date must trade the currency
pair at a specified price on the specified delivery date. Currency future contracts allow
investors to hedge against foreign exchange risk. Thus, it is being promoted by SEBI.
Hence, statement 1 is correct.

Insider trading is defined as a malpractice wherein trade of a company's securities is undertaken by


people who by virtue of their work have access to the otherwise non- public information which can

prelims.insightsonindia.com 15
© Insights Active Learning | All rights reserved - 131815. You may not reproduce, distribute or exploit the contents in any form without
written permission by copyright owner. Copyright infringers may face civil and criminal liability
Total Marks : 200
Online Prelims TEST - 10 (SUBJECT WISE)
( InsightsIAS Mock Test Series for UPSC Preliminary Exam 2020 ) Mark Scored : 98.67

be crucial for making investment decisions. Thus, it is highly discouraged by the Securities and
Exchange Board of India to promote fair trading in the market for the benefit of the common
investor. Hence, statement 2 is incorrect.

Start-up companies with a potential to grow need a certain amount of investment. Wealthy investors
like to invest their capital in such businesses with a long-term growth perspective. This capital is
known as venture capital and the investors are called venture capitalists. Thus, it is being
promoted by SEBI. Hence, statement 3 is correct.

Initial public offering (IPO) is the process by which a private company can go public by
sale of its stocks to general public. It could be a new, young company or an old company
which decides to be listed on an exchange and hence goes public. Companies can raise equity
capital with the help of an IPO by issuing new shares to the public or the existing shareholders can
sell their shares to the public without raising any fresh capital. Thus, it is being promoted by
SEBI. Hence, statement 4 is correct.
4
1 03
23 Which of the following is/are the features of Exchange Traded Funds (ETF)?4 64
1. They are cost efficient 8 75
2. They are mutual funds listed and traded on stock exchanges -
m
3. Usually, they are passive funds
i l .co
g ma
Select the correct answer using the code given below.
A. 1 only 2 2@
B. 2 and 3 only
p am
C. 1 and 3 only n u
D. 1, 2 and 3 a sa
s -d
Your Answer : D Da
am: D
Correct Answer
p
A nu
Answer Justification :

Exchange Traded Funds (ETFs) are mutual funds listed and traded on stock exchanges like
shares. Index ETFs are created by institutional investors swapping shares in an index basket, for
units in the fund. Hence, statement 2 is correct.

Usually, ETFs are passive funds where the fund manager doesn’t select stocks on your
behalf. Instead, the ETF simply copies an index and endeavours to accurately reflect its
performance. In an ETF, one can buy and sell units at prevailing market price on a real time basis
during market hours. Hence, statement 3 is correct.

ETFs are cost efficient. Given that they don’t make any stock (or security choices), they don’t use
services of star fund managers. In India, Nifty 50 and Sensex 30 ETFs charge annual expenses of
0.05 to 1 per cent of their Net Asset Value (NAV). But actively managed funds charge 2.5-3.25 per
cent a year. Hence, statement 1 is correct.

https://www.thehindubusinessline.com/opinion/columns/all-you-wanted-to-know-about-exchange-tra

prelims.insightsonindia.com 16
© Insights Active Learning | All rights reserved - 131815. You may not reproduce, distribute or exploit the contents in any form without
written permission by copyright owner. Copyright infringers may face civil and criminal liability
Total Marks : 200
Online Prelims TEST - 10 (SUBJECT WISE)
( InsightsIAS Mock Test Series for UPSC Preliminary Exam 2020 ) Mark Scored : 98.67

ded-funds/article9957174.ece

24 With reference to capital markets, consider the following statements:


1. Capital market deals in financial instruments and commodities that are long-term securities.
2. Secondary Market exclusively deals with the issue of new securities.

Which of the statements given above is/are correct?


A. 1 only
B. 2 only
C. Both 1 and 2
D. Neither 1 nor 2

Your Answer : D
Correct Answer : A

4
Answer Justification :
1 03
Capital market deals in financial instruments and commodities that4are long-term 64
5
87 statement 1 is correct.
securities. They have a maturity of at least more than one year. Hence,
-
om
The most important type of capital market is the primary.cmarket. It is what we call the new issue
market. It exclusively deals with the issue of new a il
securities, i.e. securities that are issued to
investors for the very first time. gm @
2 2
m
a capital market. This is more commonly known as
After the primary market is the secondary
u p
the stock market or the stocknexchange. Here the securities (shares, debentures, bonds, bills
etc) are bought and sold by a sainvestors.
the
s -d
D a
The main point of difference between the primary and the secondary market is that in the primary
market only new
a m securities were issued, whereas in the secondary market the trading is for already
existing u p
securities. There is no fresh issue in the secondary market. Hence, statement 2 is
An
incorrect.

25 The Financial Stability Report (FSR) is released by

A. Department of Economic Affairs


B. NITI Ayog
C. Department of Revenue
D. Reserve Bank of India

Your Answer : D
Correct Answer : D

Answer Justification :

The Reserve Bank of India today released the 19th issue of the Financial Stability Report
(FSR). The FSR reflects the collective assessment of the Sub-Committee of the Financial Stability

prelims.insightsonindia.com 17
© Insights Active Learning | All rights reserved - 131815. You may not reproduce, distribute or exploit the contents in any form without
written permission by copyright owner. Copyright infringers may face civil and criminal liability
Total Marks : 200
Online Prelims TEST - 10 (SUBJECT WISE)
( InsightsIAS Mock Test Series for UPSC Preliminary Exam 2020 ) Mark Scored : 98.67

and Development Council (FSDC) on risks to financial stability, as also the resilience of the financial
system. The Report also discusses issues relating to development and regulation of the financial
sector.

26 Which of the following sectors have been conferred with the status of Priority Sector in order to
ensure adequate institutional credit?
1. Renewable Energy
2. Export Credit
3. Social Infrastructure
4. Micro, Small and Medium Enterprises

Select the correct answer using the code given below


A. 2 and 4 only
B. 1, 2 and 3 only
C. 2, 3 and 4 only
4
03
D. 1, 2, 3 and 4
1
Your Answer : D 4 64
5
Correct Answer : D
- 87
.com
Answer Justification :
a i l
g m
http://www.arthapedia.in/index.php?title=Priority_Sector_Lending_(PSL)
2@
a m2
27 Which of the following statements best
n updescribes 'Sweat Equity Issue'?
a sa
d
A. It is issued to the -construction workers who had built the physical infrastructure of the
company. a s
B. It is onem
D
of the modes of making share based payments to employees of the company as a
u
rewardpatowards for their services.
n
C.AIt is the offer given to unemployed youth to attract them towards share market.
D. It is the offer of shares to existing shareholders as a distribution of profits.

Your Answer :
Correct Answer : B

Answer Justification :

‘Sweat equity shares’ are such equity shares, which are issued by a Company to its directors or
employees at a discount or for consideration, other than cash, for providing their know-how or
making available rights in the nature of intellectual property rights or value additions, by whatever
name called.

Sweat equity shares is one of the modes of making share based payments to employees of
the company. The issue of sweat equity shares allows the company to retain the employees
by rewarding them for their services. Sweat equity shares rewards the beneficiaries by giving
them incentives in lieu of their contribution towards the development of the company.

prelims.insightsonindia.com 18
© Insights Active Learning | All rights reserved - 131815. You may not reproduce, distribute or exploit the contents in any form without
written permission by copyright owner. Copyright infringers may face civil and criminal liability
Total Marks : 200
Online Prelims TEST - 10 (SUBJECT WISE)
( InsightsIAS Mock Test Series for UPSC Preliminary Exam 2020 ) Mark Scored : 98.67

Hence, option (b) is correct.

https://taxguru.in/company-law/issue-sweat-equity-shares.html

28 With reference to ad valorem taxes, consider the following statements.


1. It is the tax imposed on the basis of the monetary value of the taxed item
2. It is a form of progressive taxation.

Which of the statements given above is/are correct?


A. 1 only
B. 2 only
C. Both 1 and 2
D. Neither 1 nor 2

Your Answer :
4
03
Correct Answer : C
1
Answer Justification : 4 64
5
87
- value of the taxed item.
Ad valorem tax, any tax imposed on the basis of the monetary
m
.costatement 1 is correct.
Literally the term means “according to value.” Hence,
il
m a
@gconsumer spends on the taxed items. They thus
Ad valorem rates, which have come into increased use, have the important advantage of adjusting
2
m2rates against the low-priced varieties of the
the tax burden according to the amount the
a
avoid the serious discrimination of specific
p
nu form of taxation.
commodities. Thus, they are progressive
a sa
-d
https://www.britannica.com/topic/ad-valorem-tax
s
Da
am measures is/are likely to increase the taxation base in India?
29 Which of thepfollowing
u
An agriculture income into the purview of Income tax.
1. Bringing
2. Encouraging entrepreneurial activity
3. Promoting cash transactions

Select the correct answer using the code given below.


A. 2 and 3 only
B. 1 and 2 only
C. 1 only
D. 1, 2 and 3

Your Answer : B
Correct Answer : B

Answer Justification :

A tax base is the total amount of assets or revenue that a government can tax.

prelims.insightsonindia.com 19
© Insights Active Learning | All rights reserved - 131815. You may not reproduce, distribute or exploit the contents in any form without
written permission by copyright owner. Copyright infringers may face civil and criminal liability
Total Marks : 200
Online Prelims TEST - 10 (SUBJECT WISE)
( InsightsIAS Mock Test Series for UPSC Preliminary Exam 2020 ) Mark Scored : 98.67

To increase the tax base India should bring more sectors into the purview of tax regime. Thus,
bringing agriculture sector into the purview of income tax likely to increase the tax base. Hence,
statement 1 is correct.

Encouraging entrepreneurial activity will increase the profit of existing companies as well as
encourage the mushrooming of new companies. Thus, it is likely to increase the tax base. Hence,
statement 2 is correct.

Promoting cash transaction will increase the unaccounted money in the economy. Thus, lead to
increase in the black money. Therefore, it is unlikely to increase the tax base. Hence, statement 3
is incorrect.

30 With reference to debentures, consider the following statements:


1. Debenture is a movable property.
2. The debenture holders have the right to vote in meetings of the company.
4
Which of the statements given above is/are correct? 1 03
A. 1 only 4 64
5
B. 2 only
- 87
C. Both 1 and 2
D. Neither 1 nor 2 .com
a i l
Your Answer : g m
Correct Answer : A 2 2@
am
up
Answer Justification :
s an
da
A Debenture is basically- some of the loan amount the company was interested to raise from the
s
Da
public , that is why it issues debentures. A person who has bought a debenture and holding it is
m holder. A debenture holder is the creditor of the company.
called a debenture
u pa
An is a movable property. It is in the form of a certificate of indebtedness of the company
Debenture
and issued by the company itself. It generally creates a charge on the undertaking or undertakings
of the company. There is usually a specific date of redemption. Hence, statement 1 is correct.

As the debenture holders are not the owner of the company so they are not entitled with the
administration and management of the company. The debenture holders cannot claim the
privilege to vote in any meeting of the company. Hence, statement 2 is incorrect.

http://www.legalserviceindia.com/legal/article-262-concept-of-debentures-in-india.html

31 Consider the following statements


1. Reserve Bank of India keeps the cash reserve of all the scheduled banks.
2. Issue of new money is the exclusive right of Reserve Bank of India.
3. Reserve Bank of India acts as a banker to governments.

Which of the statements given above is/are correct?

prelims.insightsonindia.com 20
© Insights Active Learning | All rights reserved - 131815. You may not reproduce, distribute or exploit the contents in any form without
written permission by copyright owner. Copyright infringers may face civil and criminal liability
Total Marks : 200
Online Prelims TEST - 10 (SUBJECT WISE)
( InsightsIAS Mock Test Series for UPSC Preliminary Exam 2020 ) Mark Scored : 98.67

A. 1 and 2 only
B. 2 and 3 only
C. 1, 2 and 3
D. 1 only

Your Answer : C
Correct Answer : C

Answer Justification :

All the statements given above are correct.

It is the apex regulatory body of Indian Bank System. It keeps the cash reserves of all
scheduled banks and hence it is known as Reserve Bank.

Issue of new money is the exclusive right of Reserve Bank of India. All notes expect rupee 1
34
and coins are issued by RBI. One rupee note and coins are issued by Ministry of Finance.
410
RBI acts as a banker to governments both the centre and state. It 4 6
keeps deposits of
7 5
-8
governments and lends to governments.

. c om
il control in India?
32 Which of the following are the quantitative methods of credit
a
1. Regulation of consumer credit.
@ gm
2. Rationing Credit.
2 2
m
pa
3. Discount Rate Policy
4. Open Market Operations
n u
a sa
Select the correct answer
s - dusing the code given below
A. 1 and 2 only a
B. 1, 2 andm
D
3 only
u
C. 3 andpa4 only
n 2, 3 and 4
D.A1,

Your Answer : C
Correct Answer : C

Answer Justification :

Quantitative methods aim at controlling the cost and quantity of credit. It does not discriminate
between different sectors and end use of credit.

Quantitative methods are:

Discount Rate Policy

Open market operations

prelims.insightsonindia.com 21
© Insights Active Learning | All rights reserved - 131815. You may not reproduce, distribute or exploit the contents in any form without
written permission by copyright owner. Copyright infringers may face civil and criminal liability
Total Marks : 200
Online Prelims TEST - 10 (SUBJECT WISE)
( InsightsIAS Mock Test Series for UPSC Preliminary Exam 2020 ) Mark Scored : 98.67

Variable Reserve Ratio

Liquidity Adjustment Facility (LAF)

Marginal Standing Facility.

Regulation of Consumer Credit and Rationing of Credit are Qualitative credit control method.

33 Which of the following is/are the measures to control the inflation?


1. Increasing the GST rates on commodities
2. Increasing the transfer payments
3. Increase in government stimulus packages

4
Select the correct answers using the code given below.
1 03
A. 1 only
4 64
B. 2 only 5
C. 2 and 3 only
- 87
D. 1, 2 and 3
.com
a i l
Your Answer : A
g m
Correct Answer : A
2 2@
am
Answer Justification :
up
an
s prices of goods and services over time. Inflation is generally
d
Inflation is the increase in athe
s - Bank and/or the government. The main policy used is monetary policy –
controlled by the Central
a
D
set by Central Banks.
p am
nu in the GST rates on commodities reduce demand, leading to lower economic growth and
Increasing
A
lower inflation. Hence, statement 1 is correct.

Transfer payments are the receipts which the residents of a country get for ‘free’, without having to
provide any goods or services in return. Thus, increasing transfer payments increases spending
capacity and lead to increase in demand. Thus, it further increases inflation. Hence, statement
2 is incorrect.

Increase in government stimulus packages increases spending capacity and lead to increase in
demand. Thus, it further increases inflation. Hence, statement 3 is incorrect.

https://www.economicshelp.org/blog/2269/economics/ways-to-reduce-inflation/

34 With reference to ‘Capital Gains Tax’, which of the following statements is/are correct?
1. It is levied on long term gains only.
2. Capital gains tax in India need not be paid in case the individual inherits the property and there is
no sale.

prelims.insightsonindia.com 22
© Insights Active Learning | All rights reserved - 131815. You may not reproduce, distribute or exploit the contents in any form without
written permission by copyright owner. Copyright infringers may face civil and criminal liability
Total Marks : 200
Online Prelims TEST - 10 (SUBJECT WISE)
( InsightsIAS Mock Test Series for UPSC Preliminary Exam 2020 ) Mark Scored : 98.67

Select the correct answer using the codes given below.


A. 1 only
B. 2 only
C. Both 1 and 2
D. Neither 1 nor 2

Your Answer : B
Correct Answer : B

Answer Justification :

Capital gain can be defined as any profit that is received through the sale of a capital asset. The
profit that is received falls under the income category. Therefore, a tax needs to be paid on the
income that is received. The tax that is paid is called capital gains tax and it can either be long
term or short term. Hence, statement 1 is incorrect.

Under the Income Tax Act, capital gains tax in India need not be paid in case3the 4 individual
0
inherits the property and there is no sale. However, if the person who has
6 41inherited the
54been generated from the
property decides to sell it, tax will have to be paid on the income that has
7
- 8 leasehold rights,
sale. Some of the examples of capital assets are jewellery, machinery,
m
trademarks, patents, vehicles, house property, building, and land.
o
i l .c
Hence, statement 2 is correct. a
@ gm
2 2
https://www.bankbazaar.com/tax/capital-gains-tax.html
m
u pa
n
saPrice Index’ (WPI), consider the following statements:
35 With reference to the ‘Wholesale
a
- d commodities are taken into account while calculating WPI
1. Mandi price for agricultural
s
2. WPI basket does notacover services.
D
m
a answer using the codes given below.
u
Select the pcorrect
A.A1nonly
B. 2 only
C. Both 1 and 2
D. Neither 1 nor 2

Your Answer : C
Correct Answer : C

Answer Justification :

Wholesale Price Index (WPI) measures the average change in the prices of commodities for bulk
sale at the level of early stage of transactions. The index basket of the WPI covers commodities
falling under the three major groups namely Primary Articles, Fuel and Power and Manufactured
products. (The index basket of the present 2011-12 series has a total of 697 items
including 117 items for Primary Articles, 16 items for Fuel & Power and 564 items for
Manufactured Products.) The prices tracked are ex- factory price for manufactured products,
mandi price for agricultural commodities and ex-mines prices for minerals. Hence,

prelims.insightsonindia.com 23
© Insights Active Learning | All rights reserved - 131815. You may not reproduce, distribute or exploit the contents in any form without
written permission by copyright owner. Copyright infringers may face civil and criminal liability
Total Marks : 200
Online Prelims TEST - 10 (SUBJECT WISE)
( InsightsIAS Mock Test Series for UPSC Preliminary Exam 2020 ) Mark Scored : 98.67

statement 1 is correct.

Weights given to each commodity covered in the WPI basket is based on the value of production
adjusted for net imports. WPI basket does not cover services. Hence, statement 2 is correct.

http://www.arthapedia.in/index.php?title=Wholesale_Price_Index_(WPI)

36 Which of the following statements best describes the term ‘Tax credit’?

A. It is reward given by the government for tax payers who pay the tax in advance
B. It is a type of credit given by banks such as education loans which do not come under the
purview of taxable income
C. It is the amount of money collected from a taxpayer for a specified purpose, as tax on tax
D. It is a sum that can be subtracted from the total payable tax and offsets the overall liability.

4
Your Answer :
1 03
Correct Answer : D
4 64
5
Answer Justification : - 87
c om
il. of a person. It is basically the sum that
Tax credit is an amount that offsets the overall tax liability
a
can be subtracted from the total payable tax by an
@ gmindividual. Tax credits are different from

2 2
deductions as deductions are applicable indirectly, i.e. they help in reducing the base taxable
amount of an individual, whereas tax credits
a m directly reduce the amount of liability irrespective of
u p
the base tax liability of the tax player.
n
a sa
-d
Hence, option (d) is correct.
s
Da
https://www.bankbazaar.com/tax/tax-credit.html
p am
n u
A
37 Which of the following category of people are exempted from income tax?

A. Income of a member of a Scheduled Tribes who resides in any area in the State of Nagaland

B. Members of the Scheduled castes who resides in any area in the State of Rajasthan
C. Income of a member of a Scheduled Tribes who resides in any area in the State of Meghalaya
D. Members of the Scheduled Tribes in States of Himachal Pradesh

Your Answer :
Correct Answer : A

Answer Justification :

Income of a member of a Scheduled Tribe [as per article 366(25) of the Constitution] is exempt from
tax, if following conditions are satisfied:

prelims.insightsonindia.com 24
© Insights Active Learning | All rights reserved - 131815. You may not reproduce, distribute or exploit the contents in any form without
written permission by copyright owner. Copyright infringers may face civil and criminal liability
Total Marks : 200
Online Prelims TEST - 10 (SUBJECT WISE)
( InsightsIAS Mock Test Series for UPSC Preliminary Exam 2020 ) Mark Scored : 98.67

Such member resides in any area in the State of Nagaland, Manipur, Tripura, Arunachal
Pradesh, Mizoram or district of North Cachar Hills, Mikir Hills, Khasi Hills, Jaintia Hills and
Garo Hills or in the Ladakh region of the State of Jammu and Kashmir.

Such exemption is available in respect of income which accrues/arises from any source in
such areas or income by way of dividends/interest on securities arises from any area.

Hence, option (a) is correct.

https://taxguru.in/income-tax/all-about-tax-free-exempt-income-under-income-tax-act-1961.html

38 Consider the following statements


1. Present Cash Reserve Ratio determined by RBI is 4%
2. Present Repo rate determined by RBI is 6.5%
4
1 03
Which of the statements given above is/are correct?
4 64
A. 1 only 5
B. 2 only - 87
C. Both 1 and 2
.com
D. Neither 1 nor 2
a i l
g m
Your Answer : A
2 2@
am
Correct Answer : A
up
Answer Justification : s an
- da
s
Da
m
u pa
An

prelims.insightsonindia.com 25
© Insights Active Learning | All rights reserved - 131815. You may not reproduce, distribute or exploit the contents in any form without
written permission by copyright owner. Copyright infringers may face civil and criminal liability
Total Marks : 200
Online Prelims TEST - 10 (SUBJECT WISE)
( InsightsIAS Mock Test Series for UPSC Preliminary Exam 2020 ) Mark Scored : 98.67

4
1 03
4 64
5
- 87
.com
a i l
g m
2 2@
am of the following index?
39 Inflation in India is measured by using which
p
u
an
A. Consumers Price Index
d asfor Urban
B. Consumers Price - (Combined)
s Index
C. a
D Index
Wholesale Price
D. None a m
nup
A
Your Answer : B
Correct Answer : B

Answer Justification :

The Wholesale Price Index (WPI) was main index for measurement of inflation in India till April
2014 when RBI adopted new Consumer Price Index (CPI) (combined) as the key measure of
inflation.

40 Which among the following can result in ‘demand pull inflation’?


1. Lower interest rates on bank loans
2. Increase in subsidies
3. Increase in the cost of raw material

Select the correct answer using the codes given below.


A. 1 and 2 only

prelims.insightsonindia.com 26
© Insights Active Learning | All rights reserved - 131815. You may not reproduce, distribute or exploit the contents in any form without
written permission by copyright owner. Copyright infringers may face civil and criminal liability
Total Marks : 200
Online Prelims TEST - 10 (SUBJECT WISE)
( InsightsIAS Mock Test Series for UPSC Preliminary Exam 2020 ) Mark Scored : 98.67

B. 1 and 3 only
C. 2 and 3 only
D. 1, 2 and 3

Your Answer : A
Correct Answer : A

Answer Justification :

Demand pull inflation is caused by increased demand in the economy, without adequate increase in
supply of output. It is mainly an outcome of excess money income with the people. This high money
income would be due to increased money supply. The situation of “too much money chasing too few
goods” is an instance of demand pull inflation.

Lower interest rate on bank loans encourages people to get more loans from loans. Thus, increases
money supply in the economy which leads to increase in demand for goods. Hence, statement 1 is
correct. 34
410
Increase in subsidies increase the disposable income of the households. 4 6
Thus, it results in increased
7 5
-8
money supply in economy which leads to increase in demand for goods. Hence, statement 2 is
correct.
. c om
a il like power, labour, raw materials etc.
Cost push inflation is caused by rise in the prices of inputs

@ gm
2 raw material cost, electricity charges or wage
Price rise of inputs in the form of increased
2
m by the producer) results in increased cost and
pa
rate (including a rise in profit margin made
ultimately to increased price of theuproduct.
n
a sa
Thus, increase in the cost
s - dof raw material causes cost push inflation rather than demand pull
Da
inflation. Hence, statement 3 is incorrect.

p am
https://www.indianeconomy.net/splclassroom/what-is-the-difference-between-costpush-and-demand-
u
An
pull-inflation-how-they-can-be-tackled/

41 With reference to Minimum Alternate Tax, consider the following statements:


1. It is a direct tax imposed under Income Tax Act, 1961.
2. It does not apply to any income accruing or arising to a company from life insurance business.
3. It is not applicable to public corporate entities.

Which of the statements given above is/are correct?


A. 1 only
B. 1 and 2 only
C. 2 and 3 only
D. 1, 2 and 3

Your Answer : B
Correct Answer : B

prelims.insightsonindia.com 27
© Insights Active Learning | All rights reserved - 131815. You may not reproduce, distribute or exploit the contents in any form without
written permission by copyright owner. Copyright infringers may face civil and criminal liability
Total Marks : 200
Online Prelims TEST - 10 (SUBJECT WISE)
( InsightsIAS Mock Test Series for UPSC Preliminary Exam 2020 ) Mark Scored : 98.67

Answer Justification :

Minimum Alternate Tax (MAT) is a tax effectively introduced in India by the Finance Act of
1987, vide Section 115J of the Income Tax Act, 1961 (IT Act), to facilitate the taxation of
‘zero tax companies’ i.e., those companies which show zero or negligible income to avoid tax.
Under MAT, such companies are made liable to pay to the government, by deeming a certain
percentage of their book profit as taxable income. Hence, statement 1 is correct.

MAT is applicable to all corporate entities, whether public or private. Hence, statement 3 is
incorrect.

However, it does not apply to any income accruing or arising to a company from life
insurance business. Nor does it apply to shipping income liable to tonnage taxation as provided in
section 115V to 115VZC of the IT Act. Hence, statement 2 is correct.

http://www.arthapedia.in/index.php?title=Minimum_Alternate_Tax_(MAT)
4
1 03
42 Which of the following constitute Capital Account?
4 64
5
87
1. External Commercial Borrowings
2. Portfolio investment -
3. Bilateral loans
.com
4. Remittances a i l
@ gm
2 below:
Select the correct answer using the codes given
2
m
pa
A. 1, 2 and 3 only
B. 1, 2 and 4 only
n u
C. 2, 3 and 4 only
a sa
D. 1, 2, 3 and 4
s -d
D a
Your Answer :m A
Correct u pa : A
Answer
An
Answer Justification :

Capital Account records all international transactions of assets.

prelims.insightsonindia.com 28
© Insights Active Learning | All rights reserved - 131815. You may not reproduce, distribute or exploit the contents in any form without
written permission by copyright owner. Copyright infringers may face civil and criminal liability
Total Marks : 200
Online Prelims TEST - 10 (SUBJECT WISE)
( InsightsIAS Mock Test Series for UPSC Preliminary Exam 2020 ) Mark Scored : 98.67

0 34
6 41
54
Current Account is the record of trade in goods and services and transfer payments.
7
-8
. c om
a il
@ gm
2 2
m
u pa
n
a sa
s -d
Da
p am
u
An

Hence, option (a) is correct.

43 Which of the following can occur in economy of India due to deficit financing by the government?
1. Rise in employment rates
2. Inflation
3. Increase in money supply
4. Increased private investments

Select the correct answer using the code given below.


A. 1 and 2 only

prelims.insightsonindia.com 29
© Insights Active Learning | All rights reserved - 131815. You may not reproduce, distribute or exploit the contents in any form without
written permission by copyright owner. Copyright infringers may face civil and criminal liability
Total Marks : 200
Online Prelims TEST - 10 (SUBJECT WISE)
( InsightsIAS Mock Test Series for UPSC Preliminary Exam 2020 ) Mark Scored : 98.67

B. 1, 3 and 4 only
C. 2 and 4 only
D. 1, 2, 3 and 4

Your Answer :
Correct Answer : D

Answer Justification :

The National Planning Commission of India has defined deficit financing in the following way. The
term ‘deficit financing’ is used to denote the direct addition to gross national expenditure through
budget deficits, whether the deficits are on revenue or on capital account.

Deficit financing in India is said to occur when the Union Government’s current budget deficit is
covered by the withdrawal of cash balances of the government and by borrowing money from the
Reserve Bank of India.

0 34
Thus, in both cases, ‘new money’ comes into circulation. It is to be remembered
6 41 here that
government borrowing from the public by selling bonds is not to be considered
7 54 as deficit financing.

Hence, statement 3 is correct. -8


. c om
a ilSince deficit financing raises aggregate
It is said that deficit financing is inherently inflationary.
expenditure and, hence, increases aggregate demand,
@ gm the danger of inflation looms large. Hence,
statement 2 is correct.
2 2
pam
During inflation, private investors
n u go on investing more and more with the hope of earning
additional profits. Seeing more
a sa profits, producers would be encouraged to reinvest their
savings and accumulated
s - d profits as well as increases the employment rate. Such investment
leads to an increaseain income—thereby setting the process of economic development rolling.
D 1 and statement 4 are correct.
Hence, statementm
u pa
An
http://www.economicsdiscussion.net/public-finance/deficit-financing/deficit-financing-meaning-effec
ts-and-advantages/17460

44 With reference to Revenue Deficit, consider the following statements:


1. It is the gap between the consumption expenditure of the Government and its current revenues
2. The ‘effective revenue deficit’ does not take into account those expenditures (transfers) in the form
of grants for creation of capital assets.

Which of the statements given above is/are correct?


A. 1 only
B. 2 only
C. Both 1 and 2
D. Neither 1 nor 2

Your Answer : C
Correct Answer : A

prelims.insightsonindia.com 30
© Insights Active Learning | All rights reserved - 131815. You may not reproduce, distribute or exploit the contents in any form without
written permission by copyright owner. Copyright infringers may face civil and criminal liability
Total Marks : 200
Online Prelims TEST - 10 (SUBJECT WISE)
( InsightsIAS Mock Test Series for UPSC Preliminary Exam 2020 ) Mark Scored : 98.67

Answer Justification :

Revenue deficit is the gap between the consumption expenditure (revenue expenditure) of the
Government (Union or the State Governments) and its current revenues (revenue receipts). It also
indicates the extent to which the government has borrowed to finance the current expenditure.

Hence, statement 1 is correct.

In the Union Budget (2011-12) a new methodology of capturing the ‘effective revenue deficit’ has
been worked out, which takes into account those expenditures (transfers) in the form of grants for
creation of capital assets. Hence, statement 2 is incorrect.

http://www.arthapedia.in/index.php?title=Revenue_Deficit

45 Consider the following statements regarding Treasury Bills (T-Bills)


1. They provide long-term capital to the government.
4
03in them.
2. They satisfy the CRR and SLR requirements of the banking institutions that invest
6 41
5 4
Which of the statements given above is/are correct? 7
A. 1 only -8
B. 2 only
. c om
C. Both 1 and 2 a il
D. Neither 1 nor 2 gm @
2 2
m
pa
Your Answer : D
Correct Answer : B
n u
s a
Answer Justification :- da
D as
m they are instruments of money market used by the Central Government to fulfil its
Started in 1986,
a
up liquidity requirement upto the period of 364 days. Hence Statement 1 is
short-term
n
A
incorrect.

The TBS other than providing short-term cushion to the government, also function as short-term
investment avenues for the banks and financial institutions, besides functioning as requirements of
the CRR and SLR of the banking institutions. Hence Statement 2 is correct.

46 As per the RBI guidelines, non-performing assets is a loan or an advance where


1. Interest and/or installment of principal remain overdue for a period of more than 90 days of a term
loan.
2. The installment of principal or interest there on remains overdue for two crop seasons for short
duration crops.

Which of the statements given above is/are correct?


A. 1 only
B. 2 only
C. Both 1 and 2

prelims.insightsonindia.com 31
© Insights Active Learning | All rights reserved - 131815. You may not reproduce, distribute or exploit the contents in any form without
written permission by copyright owner. Copyright infringers may face civil and criminal liability
Total Marks : 200
Online Prelims TEST - 10 (SUBJECT WISE)
( InsightsIAS Mock Test Series for UPSC Preliminary Exam 2020 ) Mark Scored : 98.67

D. Neither 1 nor 2

Your Answer : C
Correct Answer : C

Answer Justification :

Both the statements are correct.

As per the RBI guidelines non-performing assets is a loan or an advance where

1. Interest and/or installment of principal remain overdue for a period of more than 90 days of a
term loan.

2. The bills remains overdue for a period of more than 90 days in the case of bills purchased and
4
discounted. 03
6 41
5 4
87 for two crop seasons for
3. The installment of principal or interest there on remains overdue
-
short duration crops. m o
i l .c
a
@ gm
4. The installment of principal or interest there on remains overdue for one crop season for long
duration crops. 2 2
m
u pa
n
a accounted in the Gross Domestic Product of India?
47 Which of the following servicessare
a
- d citizen in India
1. Services rendered by a foreign
s
a an Indian citizen in a foreign country.
2. Services rendered by
D
3. Income earned mby a MNC in India
u pa
Anthe correct answer using the codes given below.
Select
A. 1 and 2 only
B. 2 and 3 only
C. 1 and 3 only
D. 1 only

Your Answer : C
Correct Answer : C

Answer Justification :

Gross Domestic Product (GDP) is the total monetary or market value of all the finished goods
and services produced within a country's borders in a specific time period. As a broad measure
of overall domestic production, it functions as a comprehensive scorecard of the country’s economic
health.

Hence, statement 1 and statement 3 are correct.

prelims.insightsonindia.com 32
© Insights Active Learning | All rights reserved - 131815. You may not reproduce, distribute or exploit the contents in any form without
written permission by copyright owner. Copyright infringers may face civil and criminal liability
Total Marks : 200
Online Prelims TEST - 10 (SUBJECT WISE)
( InsightsIAS Mock Test Series for UPSC Preliminary Exam 2020 ) Mark Scored : 98.67

Gross national product (GNP) is an estimate of total value of all the final products and
services turned out in a given period by the means of production owned by a country's
residents. Hence, statement 2 is incorrect.

48 With reference to the progressive taxation system, which of the following statements is/are correct?
1. It takes a larger percentage from high-income earners than it does from low-income individuals.
2. Corporate tax is an example for progressive taxation.

Select the correct answer using the codes given below:


A. 1 only
B. 2 only
C. Both 1 and 2
D. Neither 1 nor 2

Your Answer : C
4
Correct Answer : C
1 03
4 64
Answer Justification : 5
- 87
o m
A progressive tax is a tax that imposes a lower tax rate on low-income earners compared to those

i . c
with a higher income, making it based on the taxpayer'slability to pay. That means it takes a
larger percentage from high-income earners than
g ma it does from low-income individuals.
Hence, statement 1 is correct. @ 2
m 2
pa MAT are some of the examples of progressive tax in India.
Personal income tax, corporate tax and
Hence, statement 2 is correct.nu
s a
- da
49 With reference to the a s of Industrial Production (IIP), consider the following statements:
Index
D
m year for the IIP series in India is 2004-05.
1. The current base
a
p
u and published every month by Central Statistics Office (CSO).
2. It is compiled
An has the least weightage in IIP index.
3. Electricity

Which of the statements given above is/are correct?


A. 1 only
B. 2 only
C. 2 and 3 only
D. 1, 2 and 3

Your Answer : C
Correct Answer : C

Answer Justification :

Index of Industrial Production (IIP) measures the quantum of changes in the industrial production in
an economy and captures the general level of industrial activity in the country.

The current base year for the IIP series in India is 2011-12. Hence, statement 1 is incorrect.

prelims.insightsonindia.com 33
© Insights Active Learning | All rights reserved - 131815. You may not reproduce, distribute or exploit the contents in any form without
written permission by copyright owner. Copyright infringers may face civil and criminal liability
Total Marks : 200
Online Prelims TEST - 10 (SUBJECT WISE)
( InsightsIAS Mock Test Series for UPSC Preliminary Exam 2020 ) Mark Scored : 98.67

Index of Industrial Production is compiled and published every month by Central Statistics Office
(CSO) of the Ministry of Statistics and Programme Implementation. Hence, statement 2 is
correct.

Industrial production for the purpose of IIP is divided into three sectors, i.e, Mining, Manufacturing
and Electricity. In 'Sectoral' classification, relative weights of Manufacturing, Mining and
Electricity are 75.5%, 14.2% and 10.3% respectively. Hence, statement 3 is correct.

http://www.arthapedia.in/index.php?title=Index_of_Industrial_Production

50 Which of the following form the components of the Macro-Vulnerability Index (MVI)?
1. Revenue Deficit
2. Fiscal Deficit
3. Current Account Deficit
4. Rate of Inflation
4
Select the correct answer from the codes given below: 1 03
A. 1, 2 and 3 only 4 64
5
B. 2, 3 and 4 only
- 87
C. 2 and 3 only
D. 1, 2, 3 and 4 .com
a i l
Your Answer : g m
Correct Answer : B 2 2@
am
up
Answer Justification :
s an
-
Macroeconomic Vulnerability da
Index, as mentioned in the Mid-Year Economic Analysis 2014-2015,
a s
D of Finance, adds together the rate of inflation, current account deficit
released by Ministry
a m
and fiscal deficit of a country. The Index value can be compared across countries for different
p
u to gauge their relative vulnerability. The ministry has done a comparison of the “fragile
time periods
A n
five" countries—Brazil, India, Indonesia, South Africa and Turkey.

Hence, option (b) is correct.

https://www.livemint.com/Money/zyJyTICHRC7sO3dOefWhKI/Dejargoned-Macroeconomic-Vulnerabi
lity-Index.html

51 Consider the following policy measures by the government:


1. Increasing foreign aid to underdeveloped nations
2. Increasing import duties
3. Providing export subsidies

Which of the policy measures given above may be used to reduce the Current Account Deficit
(CAD)?
A. 1 and 3 only
B. 2 and 3 only

prelims.insightsonindia.com 34
© Insights Active Learning | All rights reserved - 131815. You may not reproduce, distribute or exploit the contents in any form without
written permission by copyright owner. Copyright infringers may face civil and criminal liability
Total Marks : 200
Online Prelims TEST - 10 (SUBJECT WISE)
( InsightsIAS Mock Test Series for UPSC Preliminary Exam 2020 ) Mark Scored : 98.67

C. 1 and 3 only
D. 1, 2 and 3

Your Answer : B
Correct Answer : B

Answer Justification :

The current account measures the flow of goods, services and investments into and out of the
country. We run into a deficit if the value of the goods and services we import exceeds the
value of those we export. The current account includes net income, including interest and
dividends, and transfers, like foreign aid.

Therefore, increasing foreign aid to underdeveloped nations increases the current account deficit.
Hence, statement 1 is incorrect.

However, increasing import duties and providing export subsidies help in reducing 4
0 3 the current
account deficit. Hence, statement 2 and statement 3 are correct. 1 4
6
7 54
https://www.thehindu.com/business/Economy/what-is-current-account-deficit-and-why-does-it-matte
r/article24842306.ece - 8
o m
a il.c
mrelated to
52 Service Area Approach, sometime seen in the news, is
2 @g
A. Telecom Sector m 2
B. Banking Sector u pa
a n
C. Education Sector s
D. None of the above- da
s
Your Answer :m
A
Da
Correct u pa : B
Answer
An
Answer Justification :

Service area approach is related to lead bank scheme. Each semi-urban and rural branch allotted a
specific area to implement banking scheme.

53 Consider the following statements regarding Regional Rural Banks (RRB)


1. RRB’s were setup on the basis of the recommendations of the Narasimham Working Group.
2. The equity of RRB is held by Sponsor Bank and RBI
3. RRB’s have not been included under the ambit of priority sector lending.

Which of the statements given above is/are correct?


A. 1 and 2 only
B. 2 and 3 only
C. 1 only
D. 1, 2 and 3

prelims.insightsonindia.com 35
© Insights Active Learning | All rights reserved - 131815. You may not reproduce, distribute or exploit the contents in any form without
written permission by copyright owner. Copyright infringers may face civil and criminal liability
Total Marks : 200
Online Prelims TEST - 10 (SUBJECT WISE)
( InsightsIAS Mock Test Series for UPSC Preliminary Exam 2020 ) Mark Scored : 98.67

Your Answer : A
Correct Answer : C

Answer Justification :

Regional Rural Banks (RRBs) are financial institutions which ensure adequate credit for agriculture
and other rural sectors. Regional Rural Banks were set up on the basis of the recommendations of
the Narasimham Working Group (1975). Hence Statement 1 is correct.

The equity of a regional rural bank is held by the Central Government, concerned State Government
and the Sponsor Bank in the proportion of 50:15:35. Hence Statement 2 is incorrect.

The RRB’s have also been brought under the ambit of priority sector lending on par with the
commercial banks. Priority sector lending has been devised so that assistance from the banking
system flowed in an increasing measure to the vital sectors of the economy and according to
national priorities. Hence Statement 3 is incorrect.
4
1 03
64
54 Which of the following factors contribute to the rise of unemployment in India?
4
1. Lack of skilled labours.
8 75
2. Slowdown of the economy. -
m
3. Lack of policy support to MSMEs
i l .co
g ma
Select the correct answer from the codes given below:
A. 1 and 3 only
2 2@
m
pa
B. 2 and 3 only
C. 1 and 3 only
n u
D. 1, 2 and 3 a sa
s -d
Your Answer : D Da
am: D
Correct Answer
p
u
An Justification :
Answer

All the above given factors lead to increase in the unemployment rates.

55 Which of the following statements best describes the ‘Seed Capital’?

A. Essential capital required to procure seeds for cultivation in monsoon season.


B. It is a capital required to make initial investments in stock market.
C. Bail out capital given by the government for public sector enterprises.
D. It is an early capital required to start a new business.

Your Answer : D
Correct Answer : D

Answer Justification :

prelims.insightsonindia.com 36
© Insights Active Learning | All rights reserved - 131815. You may not reproduce, distribute or exploit the contents in any form without
written permission by copyright owner. Copyright infringers may face civil and criminal liability
Total Marks : 200
Online Prelims TEST - 10 (SUBJECT WISE)
( InsightsIAS Mock Test Series for UPSC Preliminary Exam 2020 ) Mark Scored : 98.67

Seed capital is the initial funding used to begin creating a business or a new product. Obtaining
seed capital is the first of four funding stages required for a startup to become an established
business.

A startup generally goes through four phases of investment: Seed capital, venture
capital, mezzanine funding, and an initial public offering.

Seed capital can be a relatively modest sum of money and might come from the founder's personal
assets, friends, or family. It generally covers only the first essentials such as a business plan and
initial operating expenses.

Hence, option (d) is correct.

https://www.investopedia.com/terms/s/seedcapital.asp

56 Which of the followings entities is/are allowed to participate in “call money market”?
1. Land Development Banks 0 34
2. Regional Rural Banks 6 41
4
3. Payment Banks 75
-8
Select the correct answer from the codes given below:
. c om
A. 3 only
a il
B. 1 and 2 only
@ gm
C. 1, 2 and 3
2 2
D. None am p
u
Your Answer : B s an
a
Correct Answer : A -d
D as
p am
Answer Justification :

The A numoney market (CMM) the market where overnight (one day) loans can be availed by banks
call
to meet liquidity. Banks who seeks to avail liquidity approaches the call market as borrowers and
the ones who have excess liquidity participate there as lenders. The CMM is functional from
Monday to Friday. Banks can access CMM to meet their reserve requirements (CRR and SLR) or to
cover a sudden shortfall in cash on any particular day.

Effectively, the Call Money Market is the main market oriented mechanism to meet the liquidity
requirements of banks.

Participants in the call money market are banks and related entities specified by the RBI.
Scheduled commercial banks (excluding RRBs), co-operative banks (other than Land
Development Banks) and Primary Dealers (PDs), are permitted to participate in call/notice money
market both as borrowers and lenders. As per the new regulations, Payment Banks are also
allowed to participate in CMM as both lenders and borrowers.

Hence, option (a) is correct.

prelims.insightsonindia.com 37
© Insights Active Learning | All rights reserved - 131815. You may not reproduce, distribute or exploit the contents in any form without
written permission by copyright owner. Copyright infringers may face civil and criminal liability
Total Marks : 200
Online Prelims TEST - 10 (SUBJECT WISE)
( InsightsIAS Mock Test Series for UPSC Preliminary Exam 2020 ) Mark Scored : 98.67

https://www.indianeconomy.net/splclassroom/what-is-call-money-market-cmm/

57 With reference to ‘Chit funds’, consider the following statements:


1. Registration and regulation of Chit funds are carried out by State Governments.
2. Chit funds are included in the definition of Non- Banking Financial Companies by RBI .

Which of the statements given above is/are correct?


A. 1 only
B. 2 only
C. Both 1 and 2
D. Neither 1 nor 2

Your Answer : C
Correct Answer : C

4
Answer Justification :
1 03
Chit funds are essentially saving institutions. They are of various forms and 64
4 lack any standardised
7 5
form. Chit funds have regular members who make periodical subscriptions
- 8 to the fund. The periodic
collection is given to some member of the chit funds selected m on the basis of previously agreed
c
criterion. The beneficiary is selected usually on the basis .of obids or by draw of lots or in some cases
by auction or by tender, ail m
g
2 2@
Regulatory framework am
n up
a sa under the Central Act of Chit Funds Act, 1982 and the
Chit fund business is regulated
- dAct by the various State Governments for this purpose.
Rules framed under this
D as
aofmChit funds
Central Government
p
has not framed any Rules of operation for them. Thus, Registration and

n u
Regulation are carried out by State Governments under the Rules framed by
A Hence, statement 1 is correct.
them.

Functionally, Chit funds are included in the definition of Non- Banking Financial
Companies by RBI under the sub-head miscellaneous non-banking company(MNBC). But
RBI has not laid out any separate regulatory framework for them. Hence, statement 2 is correct.

http://www.arthapedia.in/index.php?title=Chit_Funds_/_Chitty_/_Kuri/_Miscellaneous_Non-banking_
Company

58 Which of the following sectors is/are considered as Champion Sectors by Government of India?
1. Medical Value Travel
2. Audio Visual Services
3. Accounting and Finance Services

Select the correct answer using the code given below


A. 3 only

prelims.insightsonindia.com 38
© Insights Active Learning | All rights reserved - 131815. You may not reproduce, distribute or exploit the contents in any form without
written permission by copyright owner. Copyright infringers may face civil and criminal liability
Total Marks : 200
Online Prelims TEST - 10 (SUBJECT WISE)
( InsightsIAS Mock Test Series for UPSC Preliminary Exam 2020 ) Mark Scored : 98.67

B. 1 and 2 only
C. 2 and 3 only
D. 1, 2 and 3

Your Answer :
Correct Answer : D

Answer Justification :

Champion Services Sectors refers to the 12 identified sectors where the Government wants to give
focused attention for promoting their development, and realizing their potential. These include
Information Technology & Information Technology enabled Services (IT& ITeS), Tourism and
Hospitality Services, Medical Value Travel, Transport and Logistics Services, Accounting and
Finance Services, Audio Visual Services, Legal Services, Communication Services, Construction
and Related Engineering Services, Environmental Services, Financial Services and Education
Services.
4
1 03
64
Hence, option (d) is correct.
54
87 to support initiatives for
A dedicated fund of Rs. 5000 crores has been proposed to be established
-
sectoral Action Plans of the Champion Sectors. m
i l .co
a Action Plans and the implementation
The respective line Ministries/Departments finalizes the
m
timelines along with a monitoring mechanism to g monitor implementation under the overall guidance
2 @
of the Committee of Secretaries (CoS) under
m 2 Cabinet Secretary. The idea was launched and
accepted on 28 February 2018.
u pa
n
a sa
-d
59 Consider the following statements
s
1. All commercial banks
D a in India are scheduled banks
am
2. All nationalized banks in India are scheduled banks
u p
WhichAnof the statements given above is/are correct?
A. 1 only
B. 2 only
C. Both 1 and 2
D. Neither 1 nor 2

Your Answer : B
Correct Answer : B

Answer Justification :

Commercial Banks refer to both scheduled and non-scheduled commercial banks which are
regulated under Banking Regulation Act, 1949. Hence Statement 1 is incorrect.

(a) Scheduled Commercial Banks are grouped under following categories:

1. State Bank of India and its Associates

prelims.insightsonindia.com 39
© Insights Active Learning | All rights reserved - 131815. You may not reproduce, distribute or exploit the contents in any form without
written permission by copyright owner. Copyright infringers may face civil and criminal liability
Total Marks : 200
Online Prelims TEST - 10 (SUBJECT WISE)
( InsightsIAS Mock Test Series for UPSC Preliminary Exam 2020 ) Mark Scored : 98.67

2. Nationalized Banks

3. Foreign Banks

4. Regional Rural Banks

5. Other Scheduled Commercial Banks. Hence Statement 2 is correct.

A scheduled bank is a bank that is listed under the second schedule of the RBI Act, 1934. In order to
be included under this schedule of the RBI Act, banks have to fulfill certain conditions.

60 Consider the following statements regarding Consumer Food Price Index (CFPI)
1. It is a measure of change in retail prices of food products consumed by a defined population group
in a given area with reference to a base year.
2. CFPI is calculated on a monthly basis and methodology remains the same as CPI.
3. Milk and its products has highest weightage in CFPI
4
1 03
Which of the statements given above is/are correct?
4 64
5
87
A. 1 and 2 only
B. 1, 2 and 3 -
C. 2 and 3 only
.com
D. 1 only a i l
g m
Your Answer : B
2 2@
Correct Answer : A am
up
Answer Justification : s an
- da
asIndex (CFPI) is a measure of change in retail prices of food products
Consumer Food Price
D
consumed by m
a defined population group in a given area with reference to a base year. Hence
a
p 1 is correct.
Statement
u
An
The Central Statistics Office (CSO), Ministry of Statistics and Programme Implementation (MOSPI)
started releasing Consumer Food Price Indices (CFPI) for three categories -rural, urban and
combined - separately on an all India basis with effect from May, 2014. Hence Statement 2 is
correct.

Like Consumer Price Index (CPI), the CFPI is also calculated on a monthly basis and methodology
remains the same as CPI. Hence Statement 3 is incorrect.

prelims.insightsonindia.com 40
© Insights Active Learning | All rights reserved - 131815. You may not reproduce, distribute or exploit the contents in any form without
written permission by copyright owner. Copyright infringers may face civil and criminal liability
Total Marks : 200
Online Prelims TEST - 10 (SUBJECT WISE)
( InsightsIAS Mock Test Series for UPSC Preliminary Exam 2020 ) Mark Scored : 98.67

http://www.arthapedia.in/index.php?title=Consumer_Food_Price_Index_(CFPI)

0 34
6 41
61 With reference to the Global Depository Receipts, consider the following statements:
54 company.
1. It is a bank certificate issued in more than one country for shares in a7foreign
-8
2. Private markets use GDRs to raise capital denominated in euros only.
. c om
Which of the statements given above is/are correct? ail
A. 1 only
@ gm
B. 2 only 2 2
C. Both 1 and 2
pam
D. Neither 1 nor 2 n u
a sa
Your Answer :
s -d
Correct Answer : D Aa

p am
Answer u Justification :
An
A global depositary receipt (GDR) is a bank certificate issued in more than one country for shares in
a foreign company. Hence, statement 1 is correct.

It is a type of bank certificate that represents shares in a foreign company, such that a foreign
branch of an international bank then holds the shares. The shares themselves trade as domestic
shares, but, globally, various bank branches offer the shares for sale. Private markets use GDRs
to raise capital denominated in either U.S. dollars or euros. When private markets attempt to
obtain euros instead of U.S. dollars, GDRs are referred to as EDRs. Hence, statement 2 is
incorrect.

https://www.investopedia.com/terms/g/gdr.asp

62 The money accruing from the sale of “National Savings Certificates” goes to which account of the
Government of India?

prelims.insightsonindia.com 41
© Insights Active Learning | All rights reserved - 131815. You may not reproduce, distribute or exploit the contents in any form without
written permission by copyright owner. Copyright infringers may face civil and criminal liability
Total Marks : 200
Online Prelims TEST - 10 (SUBJECT WISE)
( InsightsIAS Mock Test Series for UPSC Preliminary Exam 2020 ) Mark Scored : 98.67

A. Contingency Fund of India


B. Consolidated Fund of India
C. Public Account of India
D. Prime Minister National Relief Fund

Your Answer : C
Correct Answer : C

Answer Justification :

National Savings Certificates, popularly known as NSC, is an Indian Government Savings Bond,
primarily used for small savings and income tax saving investments in India. It is part of the postal
savings system of Indian Postal Service (India Post).

These can be purchased from any Post Office in India by an adult (either in his/her own name or on
behalf of a minor), a minor, a trust, and two adults jointly. These are issued for five and ten year
maturity and can be pledged to banks as collateral for availing loans. The holder gets
0 34 the tax benefit
under Section 80C of Income Tax Act, 1961. 41
5 46
7 the government is merely
Public Account of India accounts for flows for those transactions8where
-
om
acting as a banker.

il . c
This fund was constituted under Article 266 (2) ofathe Constitution. It accounts for flows for
@
those transactions where the government is merelygmacting as a banker.
2 2
m
pa small savings and so on. These funds do not belong to
Examples of those are provident funds,
u
the government. They have to benpaid back at some time to their rightful owners. Because of this
a sa from it are not required to be approved by the Parliament.
nature of the fund, expenditures
s -d
Hence, option (c)Dais correct.
p am
u to ‘Goods and Service Tax Council (GSTC)’, consider the following statements:
An
63 With reference
1. It is a constitutional body.
2. It is headed by governor of RBI.
3. It makes recommendations to the Union and State Government on issues related to Goods and
Service Tax.

Which of the statements given above is/are correct?


A. 1 and 2 only
B. 3 only
C. 1 and 3 only
D. 1, 2 and 3

Your Answer : C
Correct Answer : C

Answer Justification :

prelims.insightsonindia.com 42
© Insights Active Learning | All rights reserved - 131815. You may not reproduce, distribute or exploit the contents in any form without
written permission by copyright owner. Copyright infringers may face civil and criminal liability
Total Marks : 200
Online Prelims TEST - 10 (SUBJECT WISE)
( InsightsIAS Mock Test Series for UPSC Preliminary Exam 2020 ) Mark Scored : 98.67

Goods & Services Tax Council is a constitutional body for making recommendations to the
Union and State Government on issues related to Goods and Service Tax. Hence, statement
1 and statement 3 are correct.

The GST Council is chaired by the Union Finance Minister and other members are the Union
State Minister of Revenue or Finance and Ministers in-charge of Finance or Taxation of all the
States. Hence, statement 2 is incorrect.

http://www.gstcouncil.gov.in/gst-council

64 Suppose a small canteen owner actually needs only two workers to carry out his business, but if his
entire family of five members are involved in running the business, then this situation is known as

A. Disguised unemployment
B. Structural unemployment
C. Cyclic unemployment 4
D. Frictional unemployment 1 03
4 64
5
Your Answer : A
- 87
Correct Answer : A
.com
Answer Justification : a i l
@ gm
2 2
Disguised Unemployment is a kind of unemployment in which there are people who are visibly
mThis situation is also known as hidden Unemployment.In
pa in a work than required.
employed but are actually unemployed.
u
such a situation more people are engaged
n
a sa
-d
Hence, option (a) is correct.
s
Da
am measures does/do reduce income inequalities?
65 Which of the following
p
nu tax slabs in income tax
1. Increased
A
2. Introduction of Direct Benefit Transfer
3. Increasing Priority Sector lending

Select the correct answer using the codes given below


A. 1 and 2 only
B. 2 only
C. 2 and 3 only
D. 1, 2 and 3

Your Answer : D
Correct Answer : D

Answer Justification :

Direct tax with increased tax slabs could be used to tax the richer population and reduces the
income inequality. Hence, statement 1 is correct.

prelims.insightsonindia.com 43
© Insights Active Learning | All rights reserved - 131815. You may not reproduce, distribute or exploit the contents in any form without
written permission by copyright owner. Copyright infringers may face civil and criminal liability
Total Marks : 200
Online Prelims TEST - 10 (SUBJECT WISE)
( InsightsIAS Mock Test Series for UPSC Preliminary Exam 2020 ) Mark Scored : 98.67

Direct benefit transfer help to reduce inequality by reducing leakages and empowering the poor.
Hence, statement 2 is correct.

Priority sector lending is tool to provide formal credit to the weaker section. Increasing Priority
sector lending will help in reducing inequality. Hence, statement 3 is correct.

66 A SWIFT Code, sometime seen in the news, is related to

A. International Banking Transaction


B. Payment Gateway System
C. Quick Toll Plaza Electronic Payment
D. None of the above

Your Answer : A
Correct Answer : A
4
1 03
64
Answer Justification :
54
- 87
The SWIFT - Society for Worldwide Interbank Financial Telecommunication - is a secure financial
message carrier — it transports messages from one bank to its intended bank recipient. Its core role
m
.co
is to provide a secure transmission channel between banks.
il
m a
g
2@ Act, 2002?
67 Which of the following is an objective of SARFAESI
a m2
A. Empowers banks to recover u p
loans.
n
sa wages for the workers.
B. Recommendation of minimum
a
d
C. Provides adequate- provisions for the implementation of priority sector lending
a
D. Rationalization s
of Union budget to increase fiscal consolidation
D
m
u pa: A
Your Answer
AnAnswer : A
Correct

Answer Justification :

The Securitisation and Reconstruction of Financial Assets and Enforcement of Securities


Interest Act, 2002 (also known as the SARFAESI Act) is an Indian law. It allows banks and other
financial institution to auction residential or commercial properties (of Defaulter) to recover loans.

The first asset reconstruction company (ARC) of India, ARCIL, was set up under this act.

https://en.wikipedia.org/wiki/Securitisation_and_Reconstruction_of_Financial_Assets_and_Enforcem
ent_of_Security_Interest_Act,_2002

68 Which of the following surveys is/are conducted by National Sample Survey Office (NSSO) once in
five years.
1. Debt and Investment

prelims.insightsonindia.com 44
© Insights Active Learning | All rights reserved - 131815. You may not reproduce, distribute or exploit the contents in any form without
written permission by copyright owner. Copyright infringers may face civil and criminal liability
Total Marks : 200
Online Prelims TEST - 10 (SUBJECT WISE)
( InsightsIAS Mock Test Series for UPSC Preliminary Exam 2020 ) Mark Scored : 98.67

2. Employment – Unemployment
3. Manufacturing Enterprises
4. Land and Livestock Holding
5. Service Sector Enterprises

Select the correct answer using the code given below


A. 2, 3 and 4 only
B. 1, 4 and 5 only
C. 2, 3, 4 and 5 only
D. 2, 3 and 5 only

Your Answer :
Correct Answer : D

Answer Justification :

4 sample
NSSO is one of the national organization which is responsible for conduct of large3scale
0
41and Programme
surveys in diverse fields on all-India basis.It comes under Ministry of Statistics
6
Implementation. 54 7
-8
NSSO conducts the surveys (once in 5 years) on
o m
o Consumer Expenditure, a il.c
m
2 @g
o Employment – Unemployment,
m 2
a
up etc.),
o Social Consumption (Health, Education
n
a sa
-d
o Manufacturing Enterprises,
s
Da
o Service Sector Enterprises
p am
nu survey of Land and Livestock Holding and Debt and Investment (once in 10
It conducts
A
years).

69 With reference to the Currency Deposit Ratio, consider the following statements
1. It shows the amount of currency that people hold as a proportion of aggregate deposits.
2. An increase in cash deposit ratio leads to an increase in money multiplier.

Which of the statements given above is/are correct?


A. 1 only
B. 2 only
C. Both 1 and 2
D. Neither 1 nor 2

Your Answer :
Correct Answer : A

prelims.insightsonindia.com 45
© Insights Active Learning | All rights reserved - 131815. You may not reproduce, distribute or exploit the contents in any form without
written permission by copyright owner. Copyright infringers may face civil and criminal liability
Total Marks : 200
Online Prelims TEST - 10 (SUBJECT WISE)
( InsightsIAS Mock Test Series for UPSC Preliminary Exam 2020 ) Mark Scored : 98.67

Answer Justification :

The currency deposit ratio shows the amount of currency that people hold as a proportion of
aggregate deposits. Hence, statement 1 is correct.

An increase in cash deposit ratio leads to a decrease in money multiplier. Hence, statement 2
is incorrect.

https://economictimes.indiatimes.com/definition/currency-deposit-ratio

70 With reference to Primary deficit, consider the following statements:


1. The primary deficit is the sum of Fiscal deficit and interest payments.
2. It includes the burden of the past debt.

Which of the statements given above is/are correct?


A. 1 only 4
B. 2 only 1 03
C. Both 1 and 2 4 64
5
D. Neither 1 nor 2
- 87
Your Answer : C .com
Correct Answer : D a i l
g m
2@
m2
Answer Justification :
a
p fiscal deficit minus net interest payments. Net
n
Gross Primary deficit is defined as ugross
a sa deficit minus net domestic lending. Hence, statement 1 is
primary deficit, is gross primary
incorrect.
s -d
Da
It excludes the
a m burden of the past debt and shows the net increase in the government’s
u p
indebtedness due to the current year’s fiscal operations. A reduction in primary deficit is
An of government’s efforts at bridging the fiscal gap during a financial year. Hence,
reflective
statement 2 is incorrect.

http://www.arthapedia.in/index.php?title=Deficit_Measurement_in_India

71 With reference to ‘Derivatives’, consider the following statements:


1. They are financial securities and are financial contracts that obtain value from underlying
securities.
2. Forward, futures, options and swaps are examples for derivatives.

Which of the statements given above is/are correct?


A. 1 only
B. 2 only
C. Both 1 and 2
D. Neither 1 nor 2

prelims.insightsonindia.com 46
© Insights Active Learning | All rights reserved - 131815. You may not reproduce, distribute or exploit the contents in any form without
written permission by copyright owner. Copyright infringers may face civil and criminal liability
Total Marks : 200
Online Prelims TEST - 10 (SUBJECT WISE)
( InsightsIAS Mock Test Series for UPSC Preliminary Exam 2020 ) Mark Scored : 98.67

Your Answer : C
Correct Answer : C

Answer Justification :

Derivatives are financial securities and are financial contracts that obtain value from
something else, known as underlying securities. Underlying securities may be stocks,
currency, commodities or bonds, etc. Hence, statement 1 is correct.

Such derivative securities or instruments are forward, futures options and swaps. Hence,
statement 2 is correct.

72 With reference to the Gender Inequality Index (GII), Consider the following statements:
1. It measures gender inequalities in three important aspects of human development—reproductive
health empowerment and economic status.
2. It is released by UNDP
0 34
3. The higher the GII value the more disparities between females and males. 1 4
6
7 54
-8
Which of the statements given above is/are correct?
A. 1 and 2 only
B. 1 and 3 only . c om
C. 2 and 3 only a il
D. 1, 2 and 3 gm @
2 2
m
Your Answer : D
u pa
Correct Answer : D
a n
s
Answer Justification :-
da
s
Da
am
The GII is an inequality
p
index. It measures gender inequalities in three important aspects of
human u development—reproductive health, measured by maternal mortality ratio and
A n
adolescent birth rates; empowerment, measured by proportion of parliamentary seats
occupied by females and proportion of adult females and males aged 25 years and older
with at least some secondary education; and economic status, expressed as labour market
participation and measured by labour force participation rate of female and male populations aged
15 years and older. Hence, statement 1 is correct.

It is released by UNDP. Hence, statement 2 is correct.

It measures the human development costs of gender inequality. Thus the higher the GII value the
more disparities between females and males and the more loss to human development. Hence,
statement 3 is correct.

http://hdr.undp.org/en/content/gender-inequality-index-gii

73 Consider the following statements regarding Indian Post Payments Bank (IPPB).
#94000

prelims.insightsonindia.com 47
© Insights Active Learning | All rights reserved - 131815. You may not reproduce, distribute or exploit the contents in any form without
written permission by copyright owner. Copyright infringers may face civil and criminal liability
Total Marks : 200
Online Prelims TEST - 10 (SUBJECT WISE)
( InsightsIAS Mock Test Series for UPSC Preliminary Exam 2020 ) Mark Scored : 98.67

1. It will be a public-private venture partially owned by the Government of India via the Department of
Posts (DoP).
2. It will focus on low-cost, low-risk, technology led solutions to extend access to formal banking.

Which of the statements given above is/are correct?


A. 1 only
B. 2 only
C. Both 1 and 2
D. Neither 1 nor 2

Your Answer : B
Correct Answer : C

Answer Justification :

Both the statements are correct.


0 34
Indian Post Payments Bank is 100% owned by the Government of India
6 41via DoP, and will
have an independent board of directors with representation from DoP and
7 54 other stakeholders from
within the Government of India to ensure strategic alignment with 8
- the overall objectives of the DoP
and the Government of India.
. c om
il
A Payments Bank is a “differentiated bank” to furtherafinancial inclusion for the underserved
gm and (ii) payments or remittance services to
population by providing (i) current and savings@accounts
2 2
migrant labour workforce, low income households, small businesses, unorganised sector entities
m
and other users.
u pa
an volume-low value transactions in deposits and payments or
shigh
a
This is to be done by enabling
d
remittance services in a-secure technology-driven environment.
s
Da
p
74 Prepaid PaymentamInstruments (PPIs) in India are regulated by
u
An
A. Securities and Exchange Board of India (SEBI)
B. Payment Settlement Authority of India (PSAI)
C. Reserve Bank of India
D. National Payment Corporation of India

Your Answer : C
Correct Answer : C

Answer Justification :

PPIs are methods that facilitate purchase of goods and services against the value stored on such
instruments.

These prepaid instruments can be issued as online wallets (e.g. paytm), mobile accounts, mobile
wallets, smart cards, magnetic stripe cards, internet accounts, paper vouchers and any such
instruments used to access the prepaid amount.

prelims.insightsonindia.com 48
© Insights Active Learning | All rights reserved - 131815. You may not reproduce, distribute or exploit the contents in any form without
written permission by copyright owner. Copyright infringers may face civil and criminal liability
Total Marks : 200
Online Prelims TEST - 10 (SUBJECT WISE)
( InsightsIAS Mock Test Series for UPSC Preliminary Exam 2020 ) Mark Scored : 98.67

https://m.rbi.org.in/scripts/FS_FAQs.aspx?Id=126&fn=9

75 Seasonal unemployment can be seen in which of the following sectors in India?


1. Farming
2. Tourism and leisure
3. Fire cracker manufacturing industries.

Select the correct answer using the code given below.


A. 1 only
B. 2 and 3 only
C. 1 and 3 only
D. 1, 2 and 3

Your Answer : D
Correct Answer : D
4
1 03
64
Answer Justification :
54
Seasonal unemployment occurs when people are unemployed at certain
- 87 times of the year, because
they work in industries where they are not needed all year round.
o m Examples of industries where
demand, production and employment are seasonal include
i l . c tourism and leisure, farming and
construction. Hence, statement 1 and statement a 2 are correct.
@ gm
2 2 work in full fledge prior to Diwali festive season and
Fire cracker manufacturing industries in India
employ more labours. However, during
p amlean season they employ less workers resulting in seasonal
u
unemployment. Hence, statement
s an 3 is correct.
- da
as Capital Fund (VCF), consider the following statements:
76 With reference to Venture
D
mFunds ensure that the money of the investors is used to fund projects which have a
1. Venture Capital
pagrow.
potentialuto
n regulated by the guidelines issued by the Reserve Bank of India (RBI)
2. TheyAare

Which of the statements given above is/are correct?


A. 1 only
B. 2 only
C. Both 1 and 2
D. Neither 1 nor 2

Your Answer : A
Correct Answer : A

Answer Justification :

Investment funds that help investors seeking private equities in startups, small, and mid-sized
enterprises possessing strong growth potential, by managing their money are known as
Venture Capital Funds (VCF). They are institutions that are dedicated to funding new ventures
and are regulated by the guidelines issued by the Securities and Exchange Board of India

prelims.insightsonindia.com 49
© Insights Active Learning | All rights reserved - 131815. You may not reproduce, distribute or exploit the contents in any form without
written permission by copyright owner. Copyright infringers may face civil and criminal liability
Total Marks : 200
Online Prelims TEST - 10 (SUBJECT WISE)
( InsightsIAS Mock Test Series for UPSC Preliminary Exam 2020 ) Mark Scored : 98.67

(SEBI). Hence, statement 2 is incorrect.

Venture Capital Funds ensure that the money of the investors is used to fund projects
which have a potential to grow and the money provided in the process is known as Venture
Capital. Hence, statement 1 is correct.

https://www.bankbazaar.com/mutual-fund/venture-capital-funds.html

77 With reference to Participatory Notes, consider the following statements:


1. The Participatory Notes holder also does not enjoy any voting rights in relation to security/shares
referenced by the Participatory Notes.
2. The investor in Participatory Notes does not own the underlying Indian security.

Select the correct answer using the code given below.


A. 1 only
B. 2 only 4
C. Both 1 and 2 1 03
D. Neither 1 nor 2 4 64
5
- 87
Your Answer : C
Correct Answer : C .com
a i l
Answer Justification : g m
2 2@
A Participatory Note (PN or P-Note)
pam
in the Indian context, in essence, is a derivative
u
an or one of its associates, against underlying Indian securities.
instrument issued in foreign jurisdictions, by a SEBI registered Foreign Institutional
a s
Investor (FII) or its sub-accounts
- d instrument may be equity, debt, derivatives or may even be an index.
The underlying Indian security
D as
The investor inmPN does not own the underlying Indian security, which is held by the FII
athe
u
who issues p PN. Thus the investors in PNs derive the economic benefits of investing in the
n
A without actually holding it. Hence, statement 2 is correct.
security

They benefit from fluctuations in the price of the underlying security since the value of the PN is
linked with the value of the underlying Indian security. The PN holder also does not enjoy any
voting rights in relation to security/shares referenced by the PN. Hence, statement 1 is
correct.

http://www.arthapedia.in/index.php?title=Participatory_Notes_(PNs)

78 Which of the following is/are included in Public Debt of India?


1. Treasury bills
2. Outstanding external debts
3. Borrowing from small savings

Select the correct answer using the code given below.


A. 1 and 2 only

prelims.insightsonindia.com 50
© Insights Active Learning | All rights reserved - 131815. You may not reproduce, distribute or exploit the contents in any form without
written permission by copyright owner. Copyright infringers may face civil and criminal liability
Total Marks : 200
Online Prelims TEST - 10 (SUBJECT WISE)
( InsightsIAS Mock Test Series for UPSC Preliminary Exam 2020 ) Mark Scored : 98.67

B. 1 and 3 only
C. 2 and 3 only
D. 1, 2 and 3

Your Answer : D
Correct Answer : A

Answer Justification :

In India, public debt refers to a part of the total borrowings by the Union Government which
includes such items as market loans, special bearer bonds, treasury bills and special loans and
securities issued by the Reserve Bank. It also includes the outstanding external debt.

However, it does not include the following items of borrowings:

(i) small savings,


4
(ii) provident funds, 1 03
4 64
5
(iii) other accounts, reserve funds and deposits.
- 87
Hence, option (a) is correct.
.com
a i l
m
@g is prepared by
79 ‘Fiscal Monitor Report’, sometime seen in the news,
2
m 2
A. World Economic Forum
u pa
n
B.
sa
International Monetary Fund
a
C.
D.
-d
European Central Bank
World Bank as
D
p :D
Your Answer am
n u
A
Correct Answer :B

Answer Justification :

The Fiscal Monitor Report surveys and analyzes the latest public finance developments. It updates
fiscal implications of the crisis and medium-term fiscal projections, and assesses policies to put
public finances on a sustainable footing. It is published by International Monetary Fund (IMF).

Other reports published by IMF are: World economic outlook, Global Financial stability report

Hence, option (b) is correct.

80 In an economy, the term for establishments having formal existence but with close to no operations
or assets is called as

A. Ghost company
B. Sick Industry

prelims.insightsonindia.com 51
© Insights Active Learning | All rights reserved - 131815. You may not reproduce, distribute or exploit the contents in any form without
written permission by copyright owner. Copyright infringers may face civil and criminal liability
Total Marks : 200
Online Prelims TEST - 10 (SUBJECT WISE)
( InsightsIAS Mock Test Series for UPSC Preliminary Exam 2020 ) Mark Scored : 98.67

C. Benami property
D. Tax haven

Your Answer : A
Correct Answer : A

Answer Justification :

A shell corporation/ Ghost company is a corporation without active business operations or


significant assets. These types of corporations are not all necessarily illegal, but they are
sometimes used illegitimately, such as to disguise business
ownership from law enforcement or the public. Shell corporations act as tax avoidance vehicles
for legitimate businesses.

Hence, option (b) is correct.

34
10
81 With reference to the Price Stabilization Fund Scheme, consider the following statements:
4
6
54
1. Fund is primarily constituted to for the purpose of containing extreme volatility in prices of selected
7
-8
commodities in the market.
2. Price stabilization fund was setup and managed by NABARD
o m
Which of the statements given above is/are correct? a il.c
A. 1 only
@ gm
B. 2 only 2 2
m
C. Both 1 and 2
u pa
D. Neither 1 nor 2 n a
s
- da
Your Answer : C s
Aa
Correct Answer : D

p am
uJustification :
An
Answer

PSFS is created and managed by government of India.

The Price Stabilization Fund (PSF) was set up in 2014-15 under the Department of Agriculture,
Cooperation & Famers Welfare (DAC&FW) to help regulate the price volatility of important agri-
horticultural commodities like onion, potatoes and pulses were also added subsequently. The PSF
scheme was transferred from DAC&FW to the Department of Consumer Affairs (DOCA) w.e.f. 1st
April, 2016. Hence Statement 2 is incorrect.

The scheme provides for maintaining a strategic buffer of aforementioned commodities for
subsequent calibrated release to moderate price volatility and discourage hoarding and
unscrupulous speculation. For building such stock, the scheme promotes direct purchase from
farmers/farmers’ association at farm gate/Mandi. Hence Statement 1 is correct.

The PSF is utilized for granting interest free advance of working capital to Central Agencies,
State/UT Governments/Agencies to undertake market intervention operations. Apart from domestic
procurement from farmers/wholesale mandis, import may also be undertaken with support from the

prelims.insightsonindia.com 52
© Insights Active Learning | All rights reserved - 131815. You may not reproduce, distribute or exploit the contents in any form without
written permission by copyright owner. Copyright infringers may face civil and criminal liability
Total Marks : 200
Online Prelims TEST - 10 (SUBJECT WISE)
( InsightsIAS Mock Test Series for UPSC Preliminary Exam 2020 ) Mark Scored : 98.67

Fund.

https://pib.gov.in/newsite/PrintRelease.aspx?relid=160050

82 With reference to Banking Codes and Standards Board of India (BCSBI), consider the following
statements
1. It was set up on the recommendation of Raghuram Rajan committee.
2. It was registered as a society under the Societies Registration Act, 1860.
3. Membership of BCSBI is compulsory to scheduled banks.

Which of the statement(s) given above is/are correct?


A. 1 and 3 only
B. 2 only
C. 2 and 3 only
D. 1, 2 and 3
4
Your Answer : 1 03
Correct Answer : B 4 64
5
- 87
Answer Justification :
.c om
a il the Committee on Procedures and
In November 2003, Reserve Bank of India (RBI) constituted
Performance Audit of Public Services under theg m
Chairmanship of Shri S.S.Tarapore (former
2 @
2
Deputy Governor) to address the issues relating to availability of adequate banking services to the
common person. am
n up
a
Membership of BCSBI is not scompulsory to scheduled banks
d a
s-
http://www.bcsbi.org.in/Abt_Background.html
D a
m
pa and ‘Save the 8th’, phrases that recently appeared in the news, are related to
83 ‘Repeal the 8th’
u
An
A. Conflict minerals in DR Congo
B. Presidential powers in Turkey
C. Women’s rights in Ireland
D. Gun control in the USA

Your Answer :
Correct Answer : C

Answer Justification :

In a historic referendum, the Irish people have voted by a landslide to repeal the 8th
amendment to the country’s constitution, allowing the government to legislate for
abortion.

The vote illustrates the monumental shift in attitudes towards women’s rights in Ireland. It’s also
testament to the power of a grassroots mobilised campaign which enabled women to share 35

prelims.insightsonindia.com 53
© Insights Active Learning | All rights reserved - 131815. You may not reproduce, distribute or exploit the contents in any form without
written permission by copyright owner. Copyright infringers may face civil and criminal liability
Total Marks : 200
Online Prelims TEST - 10 (SUBJECT WISE)
( InsightsIAS Mock Test Series for UPSC Preliminary Exam 2020 ) Mark Scored : 98.67

years’ worth of experiences of pregnancy under the 8th amendment.

http://theconversation.com/ireland-votes-to-repeal-the-8th-amendment-in-historic-abortion-referend
um-and-marks-a-huge-cultural-shift-97297

84 Consider the following statements regarding International Coral Reef Initiative (ICRI)
1. It is an informal partnership between Nations and organizations which strives to preserve coral
reefs and related ecosystems around the world.
2. The Initiative was founded in 1994 by eight governments including India.

Which of the statements given above is/are correct?


A. 1 only
B. 2 only
C. Both 1 and 2
D. Neither 1 nor 2
4
Your Answer : 1 03
Correct Answer : A 4 64
5
- 87
om
Answer Justification :

a
The International Coral Reef Initiative (ICRI) is an informal
l.c
i partnership between Nations and
organizations which strives to preserve coral reefs
@ gm and related ecosystems around the world.
Hence Statement 1 is correct. 2 2
p am
Its decisions are not binding on itsumembers.
n
a sa
- din 1994 by eight governments: Australia, France, Japan, Jamaica, the
The Initiative was founded
s
Philippines, Sweden,
D a the United Kingdom, and the United States of America. Hence Statement 2
is incorrect. m
u pa
ICRIA n
currently has more than 60 members including MoEFCCC, India.

https://www.icriforum.org/about-icri

85 Consider the following pairs:


Species Natural Habitat
1.
Grizzled Giant Squirrel Chinnar Wildlife sanctuary
2.
Gharial National Chambal sanctuary
3.
Olive Ridley Turtles Gahirmatha Marine Sanctuary

Which of the pairs given above is/are not matched correctly?


A. 1, 2 and 3
B. 2 only
C. 2 and 3 only

prelims.insightsonindia.com 54
© Insights Active Learning | All rights reserved - 131815. You may not reproduce, distribute or exploit the contents in any form without
written permission by copyright owner. Copyright infringers may face civil and criminal liability
Total Marks : 200
Online Prelims TEST - 10 (SUBJECT WISE)
( InsightsIAS Mock Test Series for UPSC Preliminary Exam 2020 ) Mark Scored : 98.67

D. None

Your Answer :
Correct Answer : D

Answer Justification :

Grizzled Giant Squirrel is living on the edge in the Chinnar Wild Life Sanctuary, one of its habitats
in Western Ghats. Researchers could count only 24 squirrels (Ratufa macroura) there, giving rise to
fears that its population might have depleted by 85% in the sanctuary over a decade.

National Chambal Sanctuary, also called the National Chambal Gharial Wildlife Sanctuary, is a
5,400 km² tri-state protected area in northern India for the protection of the Critically Endangered
gharial, the red-crowned roof turtle and the Endangered Ganges river dolphin.

Gahirmatha Marine Sanctuary is a marine wildlife sanctuary located in Odisha and is a very popular
34Ridley Turtles.
tourist attraction of Odisha in India. It is the world's largest nesting beach for Olive
0
It extends from Dhamra River mouth in the north to Brahmani river mouth in the 1 south. 4
6
7 54
86 Consider the following statements -8
1. The Tenth Schedule was inserted into Constitution in 1985co bym
52nd Amendment Act.
2. Decision of Speaker on question of disqualification is a i l .
final and cannot be challenged in the Court.
g m
3. Anti-defection law provides stability to the government by preventing shifts of party allegiance.
2 @
m 2
pa
Which of the statements given above is/are correct?
A. 2 only
n u
B. 2 and 3 only a sa
C. 1 and 3 only
s -d
D. None Da
p am
u :B
Your Answer
n
A
Correct Answer : C

Answer Justification :

What is the Anti-defection law?

The Tenth Schedule was inserted in the Constitution in 1985 by the 52nd Amendment Act.

It lays down the process by which legislators may be disqualified on grounds of defection by the
Presiding Officer of a legislature based on a petition by any other member of the House.

The decision on question as to disqualification on ground of defection is referred to the Chairman or


the Speaker of such House, and his/her decision is final and can be challenged in the court.

Advantages of anti-defection law:

Provides stability to the government by preventing shifts of party allegiance.


prelims.insightsonindia.com 55
© Insights Active Learning | All rights reserved - 131815. You may not reproduce, distribute or exploit the contents in any form without
written permission by copyright owner. Copyright infringers may face civil and criminal liability
Total Marks : 200
Online Prelims TEST - 10 (SUBJECT WISE)
( InsightsIAS Mock Test Series for UPSC Preliminary Exam 2020 ) Mark Scored : 98.67

Ensures that candidates remain loyal to the party as well the citizens voting for him.

Promotes party discipline.

https://www.insightsonindia.com/2019/09/21/insights-daily-current-affairs-pib-21-september-2019/

87 Consider the following statements regarding Export Credit Guarantee Corporation of India Limited
1. It provides export credit insurance support to Indian exporters
2. It offers guarantees to banks and financial institutions to enable exporters to obtain better facilities
from them.
3. It is under the administrative control of Ministry of Commerce and Trade

Which of the statements given above is/are correct?


A. 1 and 2 only
4
B. 2 and 3 only
1 03
C. 1 only
4 64
5
87
D. 1, 2 and 3
-
Your Answer : D
.com
Correct Answer : D
a i l
g m
Answer Justification : 2@
a m2
All the statements given above u p correct.
is/are
a n
a sExport
- d
The ECGC Limited (Formerly Credit Guarantee Corporation of India Ltd) is a company
as
wholly owned by the Government of India based in Mumbai, Maharashtra. It provides export credit
insurance supportDto Indian exporters and is controlled by the Ministry of Commerce.
p am
u
An
Functions

Provides a range of credit risk insurance covers to exporters against loss in export of goods
and services as well.

Offers guarantees to banks and financial institutions to enable exporters to obtain better
facilities from them.

Provides Overseas Investment Insurance to Indian companies investing in joint ventures


abroad in the form of equity or loan and advances.

https://en.wikipedia.org/wiki/Export_Credit_Guarantee_Corporation_of_India

88 Consider the following statements

prelims.insightsonindia.com 56
© Insights Active Learning | All rights reserved - 131815. You may not reproduce, distribute or exploit the contents in any form without
written permission by copyright owner. Copyright infringers may face civil and criminal liability
Total Marks : 200
Online Prelims TEST - 10 (SUBJECT WISE)
( InsightsIAS Mock Test Series for UPSC Preliminary Exam 2020 ) Mark Scored : 98.67

1. Mahanadi River flows through the states of Chhattisgarh and Odisha.


2. Hirakud Dam is built across Mahanadi River.

Which of the statements given above is/are correct?


A. 1 only
B. 2 only
C. Both 1 and 2
D. Neither 1 nor 2

Your Answer : C
Correct Answer : C

Answer Justification :

The Mahanadi is a major river in East Central India. It drains an area of around 141,600 square
kilometres (54,700 sq mi) and has a total course of 858 kilometres (533 mi) Mahanadi is also known
for the Hirakud Dam. 34 0
6 41
The river flows through the states of Chhattisgarh and Odisha. 5 4
7
-8
https://en.wikipedia.org/wiki/Mahanadi
o m
a il.c
89 Consider the following statements
@ gm
2 2
1. Ek Bharat Shrestha Bharat aims to actively enhance interaction between people of diverse cultures
living in different States and UTs, withathem objective of promoting greater mutual understanding
and bonding amongst them. u p
an
sinitiative
d a
2. Jigyasa is an IIT and IISc joint to develop a roadmap for research to solve major
- challenges in technology domains relevant to India.
engineering and technology
s
Da
p am
Which of the statements given above is/are correct?
u
A. 1 only
n
A
B. 2 only
C. Both 1 and 2
D. Neither 1 nor 2

Your Answer : C
Correct Answer : A

Answer Justification :

The ‘Ek Bharat Shrestha Bharat’ programme, aims to actively enhance interaction between people
of diverse cultures living in different States and UTs in India, with the objective of promoting
greater mutual understanding amongst them. As per the programme, each year, every State/UT
would be paired with another State/UT in India for reciprocal interaction between the people.
Hence Statement 1 is correct.

It is envisaged through this exchange, that the knowledge of the language, culture, traditions and
practices of different states will lead to an enhanced understanding and bonding between one

prelims.insightsonindia.com 57
© Insights Active Learning | All rights reserved - 131815. You may not reproduce, distribute or exploit the contents in any form without
written permission by copyright owner. Copyright infringers may face civil and criminal liability
Total Marks : 200
Online Prelims TEST - 10 (SUBJECT WISE)
( InsightsIAS Mock Test Series for UPSC Preliminary Exam 2020 ) Mark Scored : 98.67

another, thereby strengthening the unity and integrity of India. The States and UTs are to embark
on a mission to enhance their cultural, academic and economic ties by entering into a wide range of
mutual engagements with the paired States/UTs covering the spheres of music, drama, cuisine,
language, history, tourism and other forms of exchange between the people.

An indicative list of activities has been drawn up and circulated to the State Governments / UT
Administrations and to the key Central Ministries. The States/ UTs may choose, evolve and develop
their interaction pattern based on the suggested list as per operational suitability in the course of
their interactions.

Impacting Research Innovation and Technology (Imprint India) is an IIT and IISc joint
initiative to develop a roadmap for research to solve major engineering and technology challenges
in technology domains relevant to India. Hence Statement 2 is incorrect.

Jigyasa, a student- scientist connect programme was officially launched in the national capital
today. Council of Scientific and Industrial Research (CSIR), has joined hands with Kendriya
Vidyalaya Sangathan (KVS) to implement this programme. 4
1 03
64
The focus is on connecting school students and scientists so as to extend4student’s classroom
7 5
learning with that of a very well planned research laboratory based learning.
-8
. c om
il
90 Consider the following statements regarding Aditya L-1 Mission
a
1. It is India’s first solar mission.
@ gm
2. Satellite will be placed at 36000 km above the
2 2 surface of the Earth.
3. India aims to launch the mission by the m
p a end of 2019.
a nu is/are correct?
Which of the statements given
d as above
A. 2 only -
B. 1 and 3 only as
C. 1 only m
D
u
D. None pa
An
Your Answer : C
Correct Answer : C

Answer Justification :

The Indian Space Research Organization is planning to launch Aditya- L1 mission to study the
sun early in 2020. Hence Statement 3 is incorrect.

It is India’s first solar mission. Hence Statement 1 is correct.

It will study the sun’s outer most layers, the corona and the chromospheres and collect data about
coronal mass ejection, which will also yield information for space weather prediction.

Position of the satellite: In order to get the best science from the sun, continuous viewing of the
sun is preferred without any occultation/ eclipses and hence, Aditya- L1 satellite will be placed
in the halo orbit around the Lagrangian point 1 (L1) of the sun-earth system. Hence

prelims.insightsonindia.com 58
© Insights Active Learning | All rights reserved - 131815. You may not reproduce, distribute or exploit the contents in any form without
written permission by copyright owner. Copyright infringers may face civil and criminal liability
Total Marks : 200
Online Prelims TEST - 10 (SUBJECT WISE)
( InsightsIAS Mock Test Series for UPSC Preliminary Exam 2020 ) Mark Scored : 98.67

Statement 2 is incorrect.

https://www.insightsonindia.com/2019/09/21/insights-daily-current-affairs-pib-21-september-2019/

91 Consider the following statements


1. Etikoppaka Bommalu, toys made of wood in the region of Andhra Pradesh.
2. Kurumba Painting is traditionally practiced by tribes located in Nilgiri Hills.

Which of the statements given above is/are correct?


A. 1 only
B. 2 only
C. Both 1 and 2
D. Neither 1 nor 2

Your Answer : C
4
03
Correct Answer : C
1
Answer Justification : 4 64
5
- 87
Both the statements given above are correct.
.c om
i l
maPradesh, these toys are made with lacquer
Located on the banks of the river Varaha in Vishakhapatnam district of AP, is a small village called
Etikoppaka. Made in the Etikoppaka region of Andhra g
2
color and are traditionally known as Etikoppaka2@toys or Etikoppaka Bommalu.
pam
The village is very famous for itsn u
toys made of wood. The toys are also called as lacquer toys
s a coating
da
because of application of lacquer
-
Kurumba painting are
D asthe art form of Kurumba tribes from Nilgiris of TamilNadu. The art related to
am
the tribal rituals, describes various facets of tribal life in the forests of the Nilgiris.
u p
An
C.P.Ramaswami Aiyar Foundation in Chennai took efforts for the revival of this age old art form and
presented in to the world.

92 Commodities and Development Report 2019, sometime seen in the news, is published by

A. UN Conference on Trade and Development


B. World Trade Organisation
C. Commodity and Sensex market
D. World Economic Forum

Your Answer : A
Correct Answer : A

Answer Justification :

https://unctad.org/en/pages/PublicationWebflyer.aspx?publicationid=2499

prelims.insightsonindia.com 59
© Insights Active Learning | All rights reserved - 131815. You may not reproduce, distribute or exploit the contents in any form without
written permission by copyright owner. Copyright infringers may face civil and criminal liability
Total Marks : 200
Online Prelims TEST - 10 (SUBJECT WISE)
( InsightsIAS Mock Test Series for UPSC Preliminary Exam 2020 ) Mark Scored : 98.67

93 Which of the following are the objectives of NASA’s Parker Solar Probe?
1. To trace how energy and heat move through solar corona.
2. To explore what accelerates solar wind as well as solar energetic particles.
3. Determine structure and dynamics of magnetic fields at sources of solar wind.
4. Explore dusty plasma near Sun and its influence on solar wind and energetic particle formation.

Select the correct answer using the code given below


A. 1 and 2 only
B. 3 and 4 only
C. 1, 2, and 4 only
D. 1, 2, 3 and 4

Your Answer :
Correct Answer : D

Answer Justification : 4
1 03
All the statements given above are correct. 64
7 54
8 Probe+, abbreviated PSP) is
Parker Solar Probe (previously Solar Probe, Solar Probe Plus, or -Solar
a NASA robotic spacecraft launched in 2018 and currently en
. c om
route to probe the outer corona of the
Sun.
a il
@ gm
The Parker Solar Probe will be the first spacecraft to fly into the low solar corona. It will assess the
2
m2 plasma and magnetic field, the energy flow that heats
structure and dynamics of the Sun's coronal
a
the solar corona and impels the solarpwind, and the mechanisms that accelerate energetic particles.
a nu
d as
a s-
94 Consider the following statements regarding Pradhan Mantri Matru Vandana Yojana
D rechristened from erstwhile Indira Gandhi Matritva Sahyog Yojana
1. It is a maternity benefit
(IGMSY). a m
nup benefits are available to all Pregnant Women & Lactating Mothers.
2. The maternity
A a partial wage compensation to women for wage-loss during childbirth.
3. It provides

Which of the statements given above is/are correct?


A. 1 and 3 only
B. 1, 2 and 3
C. 3 only
D. None

Your Answer : B
Correct Answer : A

Answer Justification :

Pradhan Mantri Matru Vandana Yojana (PMMVY) is a maternity benefit rechristened from erstwhile
Indira Gandhi Matritva Sahyog Yojana (IGMSY). The IGMSY was launched in 2010.

The scheme is a conditional cash transfer schemefor pregnant and lactating women.
prelims.insightsonindia.com 60
© Insights Active Learning | All rights reserved - 131815. You may not reproduce, distribute or exploit the contents in any form without
written permission by copyright owner. Copyright infringers may face civil and criminal liability
Total Marks : 200
Online Prelims TEST - 10 (SUBJECT WISE)
( InsightsIAS Mock Test Series for UPSC Preliminary Exam 2020 ) Mark Scored : 98.67

It provides a partial wage compensationto women for wage-loss during childbirth and childcare and
to provide conditions for safe delivery and good nutrition and feeding practices.

They receive a cash benefit of Rs. 5,000 in three installments on fulfilling the respective
conditionality, viz. early registration of pregnancy, ante-natal check-up and registration of the birth
of the child and completion of first cycle of vaccination for the first living child of the family.

The eligible beneficiaries also receive cash incentive under Janani Suraksha Yojana (JSY). Thus, on
an average, a woman gets Rs. 6,000.

Exceptions: The maternity benefits are available to all Pregnant Women & Lactating
Mothers (PW&LM) except those in regular employment with the Central Government or
State Government or Public Sector Undertaking or those who are in receipt of similar benefits
under any law for the time being in force. Hence Statement 2 is incorrect.

Funding: The scheme is a Centrally Sponsored Scheme under which cost sharing ratio between the
Centre and the States & UTs with Legislature is 60:40 while for North-Eastern States
3 4 & three
10without
Himalayan States; it is 90:10. It is 100% Central assistance for Union Territories
4
Legislature. 4 6
8 75
-
https://www.insightsonindia.com/2019/09/21/insights-daily-current-affairs-pib-20-september-2019/
m
i l .co
ma
95 Consider the following statements regarding Gilgit-Baltistan
g
2@
1. It is surrounded by China, Afghanistan and India.
2
2. Indus River flows through this region. m
u pa
n
Which of the statements given
a saabove is/are correct?
A. 1 only
s -d
B. 2 only
D a
am
C. Both 1 and 2
p
D. Neither
u 1 nor 2
A n
Your Answer : C
Correct Answer : A

Answer Justification :

Where is Gilgit Baltistan located?

Located in the northern Pakistan. It borders China in the North, Afghanistan in the west, and
Tajikistan in the North West and Kashmir in the south east.

It shares a geographical boundary with Pakistan-occupied Kashmir, and India considers it as


part of the undivided Jammu and Kashmir, while Pakistan sees it as a separate from PoK.

Indus River doesn’t passes through this region.

https://www.insightsonindia.com/2019/09/21/insights-daily-current-affairs-pib-20-september-2019/

prelims.insightsonindia.com 61
© Insights Active Learning | All rights reserved - 131815. You may not reproduce, distribute or exploit the contents in any form without
written permission by copyright owner. Copyright infringers may face civil and criminal liability
Total Marks : 200
Online Prelims TEST - 10 (SUBJECT WISE)
( InsightsIAS Mock Test Series for UPSC Preliminary Exam 2020 ) Mark Scored : 98.67

96 Consider the following statements regarding Astra Missile


1. It is an indigenous beyond visual range Air-to-Air missile with a range of over 700 km.
2. It can carry nuclear warhead.
3. It is developed by ISRO

Which of the statements given above is/are correct?


A. 1 and 2 only
B. 1 only
C. 2 and 3 only
D. None

Your Answer :
Correct Answer : D

Answer Justification :
4
All the statements given above are incorrect.
1 03
64
54 of over 70 km. It has a
Astra is an indigenous Beyond Visual Range Air-to-Air missile with a range
7
15-kilogramme high-explosive pre-fragmented warhead. -8
om
.cover 5,555 kilometres per hour.
Speed: The missile can fly towards its target at a speedilof

g ma
@ Research and Development Organisation
The Astra missile has been developed by Defence
2
m2
along with 50 other public and private organisations.
a
n up
97 Consider the following pairs asa
List I
s - d List II
1.
Rhode Island Da United Kingdom
2. p am
Akademik
A nuLomonosov Aircraft carrier
3.
Rustom II Un-manned aerial vehicle

Which of the pairs given above is/are correct?


A. 1 and 2 only
B. 2 only
C. 3 only
D. 1, 2 and 3

Your Answer :
Correct Answer : C

Answer Justification :

Rhode Island, a U.S. state in New England, is known for sandy shores and seaside Colonial
towns.

prelims.insightsonindia.com 62
© Insights Active Learning | All rights reserved - 131815. You may not reproduce, distribute or exploit the contents in any form without
written permission by copyright owner. Copyright infringers may face civil and criminal liability
Total Marks : 200
Online Prelims TEST - 10 (SUBJECT WISE)
( InsightsIAS Mock Test Series for UPSC Preliminary Exam 2020 ) Mark Scored : 98.67

Akademik Lomonosov is the first floating nuclear plant to be built in the world completed its
5000-km journey.

The DRDO Rustom or TAPAS-BH-201 is a Medium Altitude Long Endurance unmanned air
vehicle being developed by Defence Research and Development Organisation for the three services,
Indian Army, Indian Navy and the Indian Air Force of the Indian Armed Forces.

98 Consider the following statements regarding Atlantic Meridional Overturning Circulation


1. It is a large system of ocean currents that carry warm water from the tropics northwards into the
North Atlantic.
2. It aids in distributing heat and energy around the earth, as the warm water it carries releases heat
into the atmosphere, and in absorbing and storing atmospheric carbon.

Which of the statements given above is/are correct?


A. 1 only
4
03
B. 2 only
1
64
C. Both 1 and 2
D. Neither 1 nor 2
5 4
- 87
om
Your Answer :
Correct Answer : C
i l .c
m a
g
2@
Answer Justification :
2
Both the statements are correct. am
n up
a
as Overturning Circulation?
What is the Atlantic Meridional

s -d
D a
The Atlantic Meridional Overturning Circulation (AMOC) is a large system of ocean
currents thatm
p a carry warm water from the tropics northwards into the North Atlantic.
u
Anin distributing heat and energy around the earth, as the warm water it carries
It aids
releases heat into the atmosphere, and in absorbing and storing atmospheric carbon.

How does the AMOC work?

The AMOC is a large system of ocean currents, like a conveyor belt, driven by differences in
temperature and salt content – the water’s density.

As warm water flows northwards it cools and some evaporation occurs, which increases the amount
of salt. Low temperature and a high salt content make the water denser, and this dense water sinks
deep into the ocean.

The cold, dense water slowly spreads southwards, several kilometres below the surface. Eventually,
it gets pulled back to the surface and warms in a process called “upwelling” and the circulation is
complete.

https://www.insightsonindia.com/2019/09/19/insights-daily-current-affairs-pib-19-september-2019/

prelims.insightsonindia.com 63
© Insights Active Learning | All rights reserved - 131815. You may not reproduce, distribute or exploit the contents in any form without
written permission by copyright owner. Copyright infringers may face civil and criminal liability
Total Marks : 200
Online Prelims TEST - 10 (SUBJECT WISE)
( InsightsIAS Mock Test Series for UPSC Preliminary Exam 2020 ) Mark Scored : 98.67

99 Global Nutrition Report, sometime seen in the news, is published by

A. United Nations
B. World Health Organisation
C. International Food Policy Research Institute
D. Food and Agricultural Organisation

Your Answer : B
Correct Answer : B

Answer Justification :

https://www.who.int/nutrition/globalnutritionreport/en/

100 Consider the following statements regarding Underground coal gasification


1. It is an industrial process which converts coal into product gas.
0 34
2. The process decomposes coal and generates carbon dioxide, hydrogen, carbon
6 41monoxide and
methane.
7 54
3. It causes minimal pollution compared to traditional coal mining - 8
om
Which of the statements given above is/are correct? il.c
A. 2 only
g ma
B. 1 and 3 only
2 2@
C. 3 only m
D. 1, 2 and 3 pa u
n
a sa
Your Answer : D
Correct Answer : D as
-d
D
m
u pa
Answer Justification :
An
All the statements given above are correct.

Underground coal gasification converts coal to gas while still in the coal seam (in-situ). Gas is
produced and extracted through wells drilled into the unmined coal seam. Injection wells
are used to supply the oxidants (air, oxygen) and steam to ignite and fuel the underground
combustion process. Separate production wells are used to bring the product gas to the surface.

The process decomposes coal and generates carbon dioxide, hydrogen, carbon monoxide
and methane.

It causes minimal environment hazard.

https://www.insightsonindia.com/2019/09/19/insights-daily-current-affairs-pib-19-september-2019/

https://en.wikipedia.org/wiki/Underground_coal_gasification

prelims.insightsonindia.com 64
© Insights Active Learning | All rights reserved - 131815. You may not reproduce, distribute or exploit the contents in any form without
written permission by copyright owner. Copyright infringers may face civil and criminal liability

You might also like